You are on page 1of 97

Common Core State Standards for High School Math:

What Every Math Teacher Should Know

ALGEBRA

Copyright 2013 by Christopher Goff, Ph.D.


FOREWORD

Thank you for choosing this book, the second in a series explaining the Common Core
State Standards in Mathematics (CCSSM) to a wide audience. The subtitle of the series
“What Every Math Teacher Should Know” is perhaps a bit over the top, but it is true
that the CCSSM have the potential to alter the K-12 mathematics education landscape
radically. And so, I offer this book as a tool for teachers to help them understand WHY
the CCSSM were written the way they were written, at least from my perspective.
Who am I, then? I’m a professional mathematician who has long been interested and
engaged with the teaching of mathematics, from K-12 to University levels. I’ve helped
out with many K-12 professional development activities, and I’ve even written a few study
guides to help California’s math teachers with their required standardized tests. In those
guides, I followed a basic approach: start with the standards (verbatim) that the teachers
need to know and explain them to the best of my ability. I have tried a similar approach
here1 . In the following pages, I list each standard and substandard verbatim, and follow
them with a series of Socratic questions, designed to lead the reader through the vocabulary
and the rationale behind each standard. I even demonstrate a few examples and solve a
few sample problems along the way. Ideally, the reader will begin to understand HOW
the CCSSM view of Algebra was devised. Understanding the bigger picture will help to
understand the mathematics.
The book follows the same structure as the CCSSM Algebra Standards, which are organized
into four sections with several subsections. The Table of Contents can be found on the next
page. Those standards that have stars (** here) fall under the “Mathematical Modeling”
umbrella, which is also one of the Standards of Mathematical Practice. In addition, those
standards with a plus (+) indicate extensions that may help students make higher-level
connections for coherence, but are not necessarily to be included on high-stakes testing.
I created all the images in the book, using either xfig2 , which is my favorite drawing
program, Geogebra3 , the Grapher program on my Mac, or occasionally even the drawing
software within LibreOffice4 . May you find in these pages what you are looking for! –CG

1
Indeed, some of this material is from the California guides, though most material is new.
2
http://xfig.org/
3
http://www.geogebra.org/cms/
4
http://www.libreoffice.org/
CONTENTS

Algebra Standard Page

SEEING STRUCTURE IN EXPRESSIONS


Interpret the structure of expressions 7
Write expressions in equivalent forms to solve problems 13

ARITHMETIC WITH POLYNOMIALS AND RATIONAL EXPRESSIONS


Perform arithmetic operations on polynomials 24
Understand the relationship between zeros and factors of polynomials 26
Use polynomial identities to solve problems 30
Rewrite rational expressions 40

CREATING EQUATIONS
Create equations that describe numbers or relationships 47

REASONING WITH EQUATIONS AND INEQUALITIES


Understand solving equations as a process of reasoning and
explain the reasoning 58
Solve equations and inequalities in one variable 62
Solve systems of equations 71
Represent and solve equations and inequalities graphically 86
ALGEBRA STANDARDS
What Every Math Teacher Should Know 7

SEEING STRUCTURE IN EXPRESSIONS

• Interpret the structure of expressions

1. Interpret expressions that represent a quantity in terms of its context.**


(a) Interpret parts of an expression, such as terms, factors, and coefficients.
– What is an expression? What is the difference between an expression, an
equation, and an identity?
In the language of algebra, expressions are the nouns (or noun clauses). They
usually represent real numbers. You can determine the value of that real
number if you “evaluate” the expression for a particular value of the variable
(also called “plugging in” a value for a variable). For instance, let’s consider
the expression x + 2. It represents a number, and you might even say that
that number is two units more than a given number x. If we evaluate the
expression at x = 3 (by plugging in 3 for x), then we obtain the number 5.
If expressions are the nouns/noun clauses of algebra, then equations (and
inequalities) are the sentences. They make statements about expressions and
numbers. These statements may be true or false, but usually we want to find
all the values for a variable that will make an equation (or inequality) into a
true statement when every expression is evaluated at the same value for the
variable. For instance, let’s consider the equation x + 2 = 5. Usually, we
want to “solve” this equation by determining all the values of the variable
that would make the statement true (i.e., x = 3 in this case).
Identities are equations (or inequalities) that are always true, regardless of
the specific values for the variables, provided all expressions are defined. For
example, x + 2x = 3x is an identity, because regardless of the value of x, the
x 1
statement is true. Similarly, x2
= x
because whenever both expressions are
defined (i.e., when x 6= 0), they are equal.
We will return to equations (and inequalities) in more detail below, in the
large sections “Creating Equations” and “Reasoning With Equations and In-
equalities.” But for now, we will focus on expressions. We will use the equals
sign to represent identities below, as we simplify expressions. We will also
8 CCSS for High School Math: Algebra - by Christopher Goff, Ph.D.

use the equals sign for when we evaluate an expression at a specific value of
a variable.

– What is the difference between terms, factors, and coefficients?


The word “term” refers to parts of an expression that are added to or sub-
tracted from each other. For instance, x2 +5x−3 has three terms: x2 , 5x, and
3. (Some books would say that terms must only be added, not subtracted.
In that case the three terms in this example would be: x2 , 5x, and −3. We
sometimes adopt this point of view.)
The word “factor” refers to parts of an expression that are multiplied together.
For instance, (x + 2)(3x2 − 7x + 8) has two factors: x + 2 and 3x2 − 7x + 8.
The word “coefficient” refers to a constant number that is being multiplied
by a power of a variable. For example, in the expression 3x2 − 7x + 8, the
coefficient of the quadratic term (x2 ) is 3 and the coefficient of the linear term
(x) is −7. In this example, the “leading” coefficient is 3 because that is the
coefficient of the highest degree term. Even if we were to write this expression
as 8 − 7x + 3x2 , we would still say that 3 is the leading coefficient.

– What are some rules for interpreting terms, factors, and coefficients?
These rules apply when using expressions to model real-world situations. If
we think of an expression as representing a number, and we want that number
to represent a quantity of something, then the expression has to have a label
on it representing that quantity (e.g. feet, kilograms, seconds, . . . ). This
means that every term in the expression has to have the same units. We
don’t allow a single number to be made up of different units: 4 feet plus 3
kilograms, for instance. It’s like the saying goes, “You can’t add apples and
oranges.” The only exception is if a conversion will bring the units together.
You can say that a quantity is 4 feet and 6 inches, for instance, but in order
to work algebraically with that quantity, you would probably have to convert
it to inches, in this case, 54 inches.
Since factors multiply together though, we have to be careful about their
units. The product of 2 feet and 3 feet is 6 square feet, for example. Multi-
plication changes the units of the answer. This means that we have to keep
track of the units of each factor in order to make sure their product has the
proper units for the answer. Coefficients are similar. Though they may look
What Every Math Teacher Should Know 9

like numbers, sometimes they really have hidden units needed to make sure
the overall term has the proper units to add to the other terms. (By the way,
physicists are really good at this kind of analysis, and some students learn to
remember physics formulas by doing an analysis of the units involved.)
– Can we do an example or two?
Sure. Let’s start with an expression that tells you how far you have traveled
if you drive 60 miles per hour (mph) for x hours: 60x. Looking at a unit
analysis:
 
60 miles
60x = (60 mph)(x hr) = (x hr) = 60x miles.
hr
When you multiply mph and hours, you get miles, which represents the dis-
tance traveled in this case. The coefficient 60 has units of mph, and x has
units of hours. Those two factors multiply together to give a term of 60x,
which has units of miles.
Let’s consider another example, to find the temperature in degrees Fahrenheit
from the temperature in degrees Celsius: 95 C + 32. Here there are two terms:
9
5
C and 32, and the first term has a coefficient of 95 . What are the units on
each term?
Since the expression has units of degrees Fahrenheit, we know that the units
on 32 are degrees Fahrenheit. The units on C are degrees Celsius. What must
9
the units on the coefficient 5
be so that this all makes sense?
Working backwards, since 59 C has units of degrees Fahrenheit and C has units
9
of degrees Celsius, we can deduce that 5
has units of degrees Fahrenheit over
degrees Celsius. Let’s check that:
9 ◦ 
5
Fahrenheit ◦ 9 ◦
◦ Celsius
(C Celsius) = C Fahrenheit.
5
But rather than just making the units work out correctly, this actually gives
us a way to interpret the fraction 59 . It involves the ratio between the sizes of
the two temperature scales. For every 1◦ change in the Celsius temperature,
9◦
the Fahrenheit temperature changes 5
. Put another way, every 5◦ Celsius
change brings about a 9◦ Fahrenheit change.
Putting it all together, we see that every part of the expression has a meaning,
and every term has units of degrees Fahrenheit. So, if we were to evaluate the
10 CCSS for High School Math: Algebra - by Christopher Goff, Ph.D.

expression at a certain value, say 40◦ Celsius for C, then we would obtain
  9 ◦ 
9 Fahrenheit
(40) + 32 = 5
◦ Celsius
(40◦ Celsius) + 32◦ Fahrenheit
5
= 72◦ Fahrenheit + 32◦ Fahrenheit
= 104◦ Fahrenheit.

That’s a pretty hot day!


Now we usually don’t track the units of each quantity at every step of the way
like we did here, but the point is that if an expression is to model a real-world
quantity, then the units need to work out correctly.
– What else could the “structure” of an expression entail?
The word “structure” could denote any aspect of the expression. For example,
often a polynomial expression is classified by its degree and/or the number
of its terms. So, x2 − 3x + 4 is a quadratic trinomial, while x − x3 is a cubic
binomial.
Another kind of structure could involve the type of functions used in the
expression. For instance, 2x is exponential, while 5 ln 2x is logarithmic.
Moreover, we have only considered expressions with one variable so far. When
more variables enter the picture, there are more possibilities for analysis.
We will look closer at the idea of “structure” in #2 below, mainly because
this concept lies at the foundation of algebra. Indeed, algebra is the study of
structure in mathematics, as well as of the rules obeyed by those structures.
The more structure students learn how to see, the better they become at
understanding algebra.
(b) Interpret complicated expressions by viewing one or more of their parts as a single
entity. For example, interpret P (1 + r)n as the product of P and a factor not
depending on P .
– How can you view parts of an expression as a single entity? Is that like
substitution?
It is like a mental version of substitution. For instance, in their example,
P (1 + r)n , if we think of x as representing (1 + r)n , then we obtain P x for the
expression. This is clearly a product of P and x. Since x does not depend on
P (x depends on r and n, though), we have a product of two factors: P , and
a factor not depending on P .
What Every Math Teacher Should Know 11

But we can also view this as a coefficient times a quantity raised to the n-th
power. If we let y represent the quantity (1 + r), then we get P y n , which is a
polynomial expression in the variable y.
– How does this information affect our interpretation of the expression?
Let’s consider this example in more detail. The expression P (1 + r)n could
represent the amount of money you have in a savings account after n years, if
the annual interest rate is r, interest is compounded annually, and your initial
deposit is P .
To verify this, we will compute how much money you have after one year and
two years, and the pattern should be clear. (Also, after zero years (n = 0), the
amount is P (1 + r)0 = P , the initial amount.) After one year, you would have
the initial amount P plus the interest, defined as P r. So your total amount
in the account is P + P r = P (1 + r).
Similarly, after another year, you have the amount that was in the account at
the beginning of that year plus the interest earned that year:

P (1 + r) + P (1 + r)r = P (1 + r)(1 + r) = P (1 + r)2 .

The pattern continues and that’s where P (1 + r)n comes from.


So now let’s return to viewing the expression P (1 + r)n as a product of P and
a factor not depending on P . This means that if you were to put some money
into a savings account for a few years, it would grow by a certain factor that
doesn’t depend on the initial deposit. Instead, that factor depends only on
the interest rate and the number of years that the money is in savings. As
a specific case, let’s say that the interest rate is 4% and we are leaving our
money there for 20 years. Then

(1 + r)n = (1 + 0.04)20 ≈ 2.1911.

This means that our initial deposit would more than double in size over those
20 years. If we put in $100, we would have about $219.11 after 20 years. If
we put in $4000, we would have about $8764.49 after 20 years. (I stored more
than four decimal places in my calculator for the latter calculation.) The final
balance in the account is proportional to the initial deposit.
2. Use the structure of an expression to identify ways to rewrite it. For example, see x4 − y 4
as (x2 )2 − (y 2 )2 , thus recognizing it as a difference of squares that can be factored as
(x2 − y 2 )(x2 + y 2 ).
12 CCSS for High School Math: Algebra - by Christopher Goff, Ph.D.

– How do you rewrite expressions?


We can use algebraic identities to rewrite expressions, because identities are al-
ways equal, regardless of what values you plug in for the variables. So, in their
example, x4 −y 4 is equal to (x2 −y 2 )(x2 +y 2 ). Indeed, this is an algebraic identity.
To check it, we can multiply out the factored form.

(x2 − y 2 )(x2 + y 2 ) = x4 + x2 y 2 − x2 y 2 − y 4 = x4 − y 4 .

But the point here is that we can factor a difference of two fourth powers by
identifying it as a difference of two squares first, and then using the pattern for
factoring a difference of two squares. Of course, you can also notice that one of the
factors is itself a difference of two squares, and can thus be factored. Therefore,
x4 − y 4 = (x − y)(x + y)(x2 + y 2 ).
A similar example is to factor the expression x6 + 5x3 + 6. Here, we can mentally
view x3 as “the variable” (say y) and then the expression becomes y 2 + 5y + 6,
which can be factored (y + 2)(y + 3). So the original expression can be factored
as (x3 + 2)(x3 + 3). Depending on how far you want to go, you can either stop
√ √
here, or you can factor each of these as a “sum of two cubes,” using 3 2 and 3 3
as the cube roots of 2 and 3. Thus,
√ √ √ √ √ √
x6 + 5x3 + 6 = (x + 2)(x2 − x 2 + 4)(x + 3)(x2 − x 3 + 9)
3 3 3 3 3 3

– Why would you want to rewrite expressions?


A quick answer is that rewriting an expression allows you to have yet another
way to interpret it. We saw above that the structure of an expression can aid in
its interpretation, especially in real-world situations. So, being able to rewrite an
expression may lead to more insight into its meaning.
As an example, suppose that your phone company told you that from now on,
your monthly bill would be 32.5(1 + 0.02m + 0.06t) dollars, if you talked for m
minutes and sent t text messages. As it is written, it would be hard for you to
determine the base cost per month, the cost per minute, and the cost per text
message. But if you multiply out the expression to get 32.5 + 0.65m + 1.95t, then
it is easy to see that you would have to pay $32.50 per month as a base rate, plus
65 cents per minute, plus $1.95 per text message. (And then you would probably
look for a cheaper plan!)
What Every Math Teacher Should Know 13

Rewriting expressions can also be a useful tool to solve problems. This is covered
more extensively in the next section.
A deeper answer to this question is that the better students are at seeing an
expression from multiple perspectives, the better they will understand algebra.
What may seem like “tricks” to some students are often just looking at an ex-
pression from a different point of view. This skill is being explicitly highlighted in
these Common Core standards, with the goal of deepening student understanding
of algebraic structures.

• Write expressions in equivalent forms to solve problems

3. Choose and produce an equivalent form of an expression to reveal and explain properties
of the quantity represented by the expression.**
(a) Factor a quadratic expression to reveal the zeros of the function it defines.
– What are zeros again?
A zero of a polynomial expression is a value of the variable that makes the
expression evaluate to zero. They are also called “roots” of the polynomial.
Recall that a degree n polynomial has at most n distinct roots. So for quadrat-
ics, we are looking for at most two distinct zeros.
– Why do we have to factor the expression to find its zeros? Is there another
way?
The reason that factoring will tell you the zeros has to do with multiplication
properties of the number zero. Certainly it is true that if x = 0 or y = 0,
then xy = 0. Zero multiplied by any number is zero. For real numbers (or
complex numbers, or rational numbers, or any other “field” of numbers), the
converse of this property is true also. So if xy = 0, then it has to be true that
either x = 0 or y = 0. This property is so useful that it is exploited often as
a way to solve polynomial equations.
But it is not the only way to find zeros. Sometimes you can tell by inspec-
tion, which means just by looking at the expression. Other times, factoring a
polynomial might prove too difficult, or perhaps you have decimal coefficients
that arise from data taken in an experiment. In these cases, you can approx-
imate the zeros by using the quadratic formula. More generally, you can try
graphing the polynomial. This is covered below, beginning on page 86.
14 CCSS for High School Math: Algebra - by Christopher Goff, Ph.D.

– What are some examples?


First, let’s consider x2 + x, which can be factored as x(x + 1). Therefore the
zeros will be those numbers that make either factor equal to zero. So x = 0
and x = −1 are the two zeros of x2 + x. You could have probably figured out
that x = 0 was one of the zeros by inspection, but finding the other one may
have taken longer.
Second, consider x2 −5x−6, which equals (x−6)(x+1). Its zeros are 6 and −1.
To check: 62 −(5)(6)−6 = 36−30−6 = 0 and (−1)2 −5(−1)−6 = 1+5−6 = 0.
Next, let’s look at x2 − 2x + 1, which can be factored as (x − 1)2 . Therefore,
x = 1 is the only zero of this expression, and it is a “double root.” This will
play a role in graphing, as we will see later.
Sometimes a quadratic polynomial doesn’t factor, like x2 + x + 1. In this case,
there are no zeros in the real numbers. (We will be able to find its complex
zeros later on, using the quadratic formula.)
(b) Complete the square in a quadratic expression to reveal the maximum or minimum
value of the function it defines.
– What is completing the square again? And why does completing the square
tell you the maximum or minimum value of the expression?
Rather than explain completing the square in the usual way (which you can
find in many textbooks and online), let’s look at this situation in a new way,
one outlined in the Common Core Standards.
But first, let’s just consider the simple quadratic expression x2 . It is already
factored, and zero is a double root. What are the maximum and minimum
values of x2 ? If you mentally substitute some values for x, you will soon see
that there is no maximum value for the expression. Plugging in larger and
larger positive values of x shows that x2 grows without bound. But there is
a minimum value. If x = 0, then x2 = 0. This is the minimum value of x2 .
Now let’s throw a coefficient in there. What about the expression −x2 ? (This
is NOT (−x)2 , by the way.) Does it have a maximum value? A minimum
value? If you think about it, and possibly plug in some values, you will see
that −x2 has a maximum value of 0 at x = 0, and it does not have a minimum
value. What about 3x2 ? Since the minimum value of x2 is zero at x = 0, the
minimum value of 3x2 is 3(0) = 0, and it also happens when x = 0.
What about adding (or subtracting) a constant, like say x2 +5? It’s clear that
What Every Math Teacher Should Know 15

x2 + 5 will reach a minimum when x2 reaches its minimum. That minimum


value will be increased by the constant amount of 5, but it will still be a
minimum. When x = 0, the minimum value of x2 + 5 is 5, and there is no
maximum value. Similarly, −x2 − 3 has a maximum value of −3 at x = 0,
and no minimum value.
If we throw in a linear term, though, it gets trickier. What about x2 + 4x?
Does it have a maximum value? A minimum value? If so, what are they?
Here’s where completing the square comes in. One way to interpret completing
the square is that we are making a substitution that removes the linear term.
Then, our expression will look like ay 2 + b. We know from what we did above
that if a is positive, then ay 2 + b has a minimum value of b at the point y = 0,
and if a is negative, then ay 2 + b has a maximum value of b at the point y = 0.
So, completing the square reduces a more complicated quadratic expression
to a simpler one.
– What are some examples?
Before we do an example, let’s look at the pattern obtained by squaring the
binomial x + b:
(x + b)2 = x2 + 2bx + b2 .

So if we have something of the form x2 + ax, then in order for it to be a


square, we will need to halve the coefficient a to find b.
Let’s start with x2 + 4x. Because the coefficient on x2 is 1 and the coefficient
of x is 4, we know that completing the square should give us an (x + 2)2 term
(because half of 4 is 2). Following the pattern, (x + 2)2 = x2 + 4x + 4. So, by
adding and subtracting 4 from the original expression, we obtain

x2 + 4x = x2 + 4x + 4 − 4 = (x + 2)2 − 4.

This is in the form y 2 − 4, if we let y represent x + 2. So we know it has a


minimum value of −4 that occurs when y = 0, i.e., when x = −2. We can
check that by evaluating the expression at x = −2: (−2)2 + 4(−2) = 4 − 8 =
−4.
What is the minimum value of 2x2 + 5x + 5? I usually start by factoring out
the leading coefficient: 2(x2 + 52 x + 25 ). Yes, this introduces fractions, but
there’s no way around that now. Remember that we do not have an equation
16 CCSS for High School Math: Algebra - by Christopher Goff, Ph.D.

here, and so we can’t “multiply both sides” of the equation by 2 or anything


5
like that. Since half of 2
is 54 , and squaring that gives us 25
16
, I will add and
25
subtract 16
strategically inside the parentheses.
 
2 2 5 5
2x + 5x + 5 = 2 x + x +
2 2
 
2 5 25 5 25
= 2 x + x+ + −
2 16 2 16
 2 !
5 15
= 2 x+ +
4 16
 2
5 15
= 2 x+ +
4 8
In the last step, I multiplied through by 2, the leading coefficient. So the
15
minimum value of the expression is 8
and it occurs when x = − 45 .
This may not be the way that you were taught how to complete the square,
nor am I saying that you should be doing it this way. I do not want to
prescribe one method at the expense of other acceptable solutions. Rather, I
hope you will find your own way that works for you. As long as you follow
algebraic rules, your solution method should be valid.
Now let’s consider −3x2 + 9x − 10, which has a maximum value and no min-
imum value. We will find that maximum value and where it occurs. Rather
than explain in words, I will just write out the step by step equations below.
 
2 2 10
−3x + 9x − 10 = −3 x − 3x +
3
 
2 9 10 9
= −3 x − 3x + + −
4 3 4
 2 !
3 13
= −3 x− +
2 12
 2
3 13
= −3 x − −
2 4

So, the maximum value of the expression is − 13


4
, and it occurs when x = 32 .
(c) Use the properties of exponents to transform expressions for exponential functions.
For example the expression 1.15t can be rewritten as (1.151/12 )12t ≈ 1.01212t to
reveal the approximate equivalent monthly interest rate if the annual rate is 15%.
– What are the properties of exponents again?
What Every Math Teacher Should Know 17

Probably the most important ones are:


i. xn xm = xn+m
ii. xn y n = (xy)n
iii. (xn )m = xnm
for any values of m and n, provided that x and y are positive numbers.
– What do they mean in the example:

1.15t = (1.151/12 )12t ≈ 1.01212t ?

This example applies the third property above in a novel way, by writing the
1
exponent t as a product of two factors: 12
and 12t. The reason for this is
because it now ties the formula for annually compounded interest, P (1 + r)t ,
with that of monthly compounded interest, P (1 + m)12t . Here, r is the annual
interest rate, and m is the monthly interest rate.
So, we interpret 1.15t as (1+0.15)t , which means that we have an annual inter-
est rate of 15%. But the approximately equivalent expression, (1 + 0.012)12t ,
tells us that the approximate monthly interest rate is 1.2%. Because of the
mathematical effects of compounding interest, this interest rate is not equal
to dividing the annual interest rate by 12 (though it is often close). Let’s
be more precise. Since (1.15)1/12 ≈ 1.011715, an annual interest rate of 15%
corresponds to a monthly interest rate of about 1.1715% (≈ 1.2%), rather
1
than 12
(15%) = 1.25%.
– What are some other ways to transform exponential expressions?
One related example is that you can change the base of an exponential ex-
pression to almost any other base. For example, if we wanted to write 2x as
a power of 10, we could (using logarithms) write

2x = (10log 2 )x = 10x log 2 ≈ 100.301x .

So, if you wanted to know how many digits 2100 has, you could first convert
2100 ≈ 1030.1 . Then you could use more properties of exponents to write
1030.1 = 100.1 1030 ≈ 1.2589 × 1030 . So 2100 has 31 digits.
An another example of this 2x approximation comes in computers. Notice that
1024 = 210 ≈ 103.01 ≈ 103 = 1000. The word “kilobyte” used to mean 1024
bytes (because computers deal a lot with binary arithmetic) but apparently
18 CCSS for High School Math: Algebra - by Christopher Goff, Ph.D.

now it more commonly means 1000 bytes, making it more consistent with the
meaning of “kilo” in the metric system.

4. Derive the formula for the sum of a finite geometric series (when the common ratio is not
1), and use the formula to solve problems. For example, calculate mortgage payments.**

– The terms “sequence” and “series” are familiar. What is the difference between
a “sequence” and a “series”? What does “finite” mean?
I like to think of a sequence as a list of items called “terms”. Usually, a sequence is
represented as a list of numbers, separated by commas. A series, to the contrary,
represents the sum of the terms of the sequence. Here, “finite” means that the
list of terms contains only finitely many numbers. Thus, we should be able to
find the sum for the finite series that we consider. If a sequence is infinite, then
we have to determine if the infinite sum of its terms even makes sense. We will
leave the topic of infinite series for students and teachers of Calculus and beyond,
and instead we will deal only with finite series here.

– The terms “arithmetic” and “geometric” often refer to sequences and series. What
is the difference between an “arithmetic” sequence and a “geometric” sequence?
A sequence is “arithmetic” if there is a common difference between successive
terms. For example, the sequence 1, 4, 7, 10, 13, 16, . . . is arithmetic because
there is a common difference of 3 between successive terms. The formula for the
n-th term of an arithmetic sequence is linear: an = a1 + (n − 1)d, where d is the
common difference and a1 is the first term.
A sequence is “geometric” if there is a common ratio between successive terms.
For example, the sequence 21 , 1, 2, 4, 8, . . . is geometric because there is a common
ratio of 2 between successive terms. The formula for the n-th term of a geometric
sequence is exponential: an = a1 rn−1 , where r is the common ratio and a1 is the
first term.

– How do you derive the formula for the sum of a finite geometric sequence?
1
Let’s do an example first. Suppose we want to add up 2
+ 1 + 2 + 4 + 8 using
a shortcut. We begin by multiplying the series by 2 to get another series that
has many of the same terms. We then cancel most of those terms by subtracting
this new series from the old one. Then we see what remains. This is written out
explicitly below.
What Every Math Teacher Should Know 19

1
2
+1 +2 +4 +8 = S
−(1 +2 +4 +8 + 16 = 2S)
1
2
+0 + 0 + 0 − 16 = −S
1
− 16
2 31
And so S = = . Repeating this in general gives
−1 2
a1 − a1 r n
Sn = a1 + a1 r + . . . + a1 rn−1 = .
1−r
One of the Sample Problems below asks the reader to fill in the missing steps
in this derivation. Notice that if r = 1, we cannot use this approach, because
the denominator would be zero. But if r = 1, then the sequence is made up of
constant terms, and the corresponding value of the series can thus be found by
multiplication.
– What is a mortgage payment? How do you use this formula to calculate mortgage
payments?
Most people who want to buy a house cannot afford to pay the full price of the
house all at once. So, most house buyers take out a mortgage (a loan from a
bank) and they pay off this loan each month. That monthly payment is called
a mortgage payment. If the bank didn’t charge interest, then calculating the
mortgage payment would be easy: simply divide the cost of the house by the
number of payments you would make. For example, a 30-year mortgage would
require 360 monthly payments. So, a $100,000 house could be paid off over 360
months for just
100,000
≈ $277.78 per month.
360
But banks charge interest. (We will ignore other complications, like down pay-
ments, etc., here.) And so the calculations get more involved. Let’s say that you
take out a house loan for H dollars and that the annual interest rate is r. Suppose
your monthly payment is M dollars, over Y total years of the loan. (That means
that there are 12Y total payments.) How much should each payment be? In other
words, how does M depend on H, r, and Y ? Let’s calculate how much you owe
each month, and we’ll look for a pattern.
The starting value of the loan is H. This is how much you owe in month zero.
Then, in month 1, the interest has been added to the loan, but you have also
r r
made one payment. So, you still owe H + 12 H − M = (1 + 12 )H − M on the loan
after one month.
20 CCSS for High School Math: Algebra - by Christopher Goff, Ph.D.

After two months, you owe last month’s amount plus the interest, but you have
made another payment. So, you owe
 r  h r i  r 2  r
1+ 1+ H −M −M = 1+ H − 1+ M −M
12 12 12 12
r
Continuing the pattern, and setting (1 + 12
) = R, after three months, you will
owe
R3 H − R2 M − RM − M.

So, after a typical 30-year loan (360 months), you will owe

R360 H − R359 M − R358 M − . . . − R2 M − RM − M.

If you paid off your loan, then this amount should be equal to zero! So that means
that the only positive term, R360 H, should equal the sum of the negative terms,
R359 M + . . . + RM + M , which is a geometric series that can be summed using
the formula. Putting it all together, we get:

M − R360 M 1 − R360
 
360
R H= =M .
1−R 1−R

Solving for M , we obtain


 
360 1−R
M =R H
1 − R360

As an example, say you got a 30-year loan for $100,000 at 6% interest. So H =


0.06
100,000, and R = (1 + )
= 1.005. Then the monthly payment would be
12
 
360 1 − 1.005
M = (1.005) (100,000) ≈ 599.55,
1 − (1.005)360

or $599.55 each month. Notice that this is more than twice what you would pay
if there were no interest.
– Sample Problems
(a) Complete the square to find extreme (maximum or minimum) values for each
quadratic expression. State where each extreme value occurs and what the
extreme value is.
i. x2 − 10x.
ii. −3x2 + 9x.
What Every Math Teacher Should Know 21

iii. 5x2 + 10x + 25.


(b) Using 2 ≈ 100.301 , find the number of digits in 2500 and 444 .
(c) Find the sum of the following series.
4 4 4
i. 36 + 12 + 4 + + +
3 9 27
1 1 1 1
ii. + + +
2 4 8 16
2 4 8 16
iii. + + +
3 9 27 81
iv. 1 − 1 + 1 − 1 + . . . + (−1)n
v. 1 + 1.1 + 1.21 + 1.331 + 1.4641 + . . . + (1.1)10
a1 − a1 rn
(d) Fill in the details to derive the geometric sum formula Sn = .
1−r
(e) Calculate the monthly payments for the same loan above (30 years, $100,000),
but with an interest rate of 5.4%. How much do you save each month com-
pared to the monthly payment at 6%?
(f) Suppose you want to take out a car loan of $20,000 for 5 years at 4.8% interest.
Calculate your monthly payment.
(g) Suppose you want to take out a loan of L dollars for Y years at an interest
rate of r. Calculate your monthly payment M as a function of L, Y , and r.
– Answers to Sample Problems
(a) Complete the square to find extreme (maximum or minimum) values for each
quadratic expression. State where each extreme value occurs and what the
extreme value is.
i. x2 − 10x. x2 − 10x = (x − 5)2 − 25.
So, x2 − 10x has a minimum value of −25 when x = 5.
 2
2 2 3 27
ii. −3x + 9x. −3x + 9x = −3 x − + .
2 4
27 3
So, −3x2 + 9x has a maximum value of when x = .
4 2
iii. 5x2 + 10x + 25. 5x2 + 10x + 25 = 5(x + 1)2 + 20.
So, 5x2 + 10x + 25 has a minimum value of 20 when x = −1.
(b) Using 2 ≈ 100.301 , find the number of digits in 2500 and 444 .
Since 2500 ≈ 10500(0.301) ≈ 10150.5 , 2500 has 151 digits.
Similarly, 444 = 288 ≈ 1088(0.301) ≈ 1026.488 , and so 444 has 27 digits.
(c) Find the sum of the following series.
22 CCSS for High School Math: Algebra - by Christopher Goff, Ph.D.

4
4 4 4 36 − 81 1456
i. 36 + 12 + 4 + + + = 1 =
3 9 27 1− 3 27
1 1
1 1 1 1 − 15
ii. + + + = 2 32 1 =
2 4 8 16 1− 2 16
2 32
2 4 8 16 − 130
iii. + + + = 3 243 2 =
3 9 27 81 1− 3 81
n
iv. 1 − 1 + 1 − 1 + . . . + (−1) . It depends. If n is even, then the sum is 1. If
n is odd, then the sum is 0. Notice that this series starts at n = 0.
1 − (1.1)11
v. 1+1.1+1.21+1.331+1.4641+. . .+(1.1)10 = = 10[(1.1)11 −1] ≈
1 − 1.1
18.53 (using a calculator)
a1 − a1 r n
(d) Fill in the details to derive the geometric sum formula Sn = .
1−r
a1 +a1 r + . . . +a1 rn−1 = Sn
−(a1 r +a1 r2 + . . . +a1 rn = rSn )
a1 +0 + . . . −a1 rn = (1 − r)Sn
a1 − a1 r n
Therefore, Sn = .
1−r
(e) Calculate the monthly payments for the same house loan above (30 years,
$100,000), but with an interest rate of 5.4%. How much do you save each
month compared to the monthly payment at 6%?
0.054
The only thing that changes in this formula is that R is now (1 + 12
) =
1.0045. So, the monthly payment is
 
360 1 − 1.0045
M = (1.0045) (100,000) ≈ 561.53,
1 − (1.0045)360
or $561.53 each month. This is a savings of about $38.02 each month, which
may not seem like a big difference, but when multiplied by 360 months, it
means that you are saving over $13,600 over the life of the loan!
(f) Suppose you want to take out a car loan of $20,000 for 5 years at 4.8% interest.
Calculate your monthly payment.
Here, we need to change the number of months of the loan (from 360 to 60),
the amount of the loan (from 100,000 to 20,000), and the interest rate. So
0.048
the new R will be (1 + 12
)
= 1.004. Thus the monthly payment will be
 
60 1 − 1.004
M = (1.004) (20,000) ≈ 375.59,
1 − (1.004)60
or $375.59 each month.
What Every Math Teacher Should Know 23

(g) Suppose you want to take out a loan of L dollars for Y years at an interest
rate of r. Calculate your monthly payment M as a function of L, Y , and r.
It’s probably good practice to go through the entire derivation again, but if
you do, you should obtain:
r
− 12
 
r 12Y

M =L 1+ r 12Y .
12 1 − (1 + 12 )
r
If we let R = (1 + 12
), then the formula may look simpler:
 
12Y 1−R
M = LR .
1 − R12Y
24 CCSS for High School Math: Algebra - by Christopher Goff, Ph.D.

ARITHMETIC WITH POLYNOMIALS


AND RATIONAL EXPRESSIONS

• Perform arithmetic operations on polynomials

1. Understand that polynomials form a system analogous to the integers, namely, they are
closed under the operations of addition, subtraction, and multiplication; add, subtract,
and multiply polynomials.

– What kind of a system is analogous to the integers?


The properties listed here (along with a few more) define a class of objects in
modern abstract algebra called a ring. Rings have been studied abstractly for
over a hundred years.
– What is a ring? What are some examples?
A ring is a set R together with two operations (called + and ×) satisfying the
following properties. Suppose that a, b, and c are elements of R.
(a) R is closed under + and ×
This means that a + b and a × b are elements of R. [The element a × b is often
written ab. We will follow this conventional notation below.]
(b) + and × are associative
This means that (a + b) + c = a + (b + c) and (ab)c = a(bc).
(c) + and × are commutative
This means that a + b = b + a and ab = ba.
(d) + and × have identity elements 0 and 1 in R, respectively
This means that a + 0 = a and b · 1 = b.
(e) every element has an additive inverse in R
This means that for every a in R, there is an element −a in R satisfying
a + (−a) = 0.
(f) × distributes over +
This means that a(b + c) = ab + ac.
The most common rings we use are the integers, the rational numbers, the real
numbers, and the complex numbers. But rings don’t have to just be made up
What Every Math Teacher Should Know 25

of numbers. For instance, the set of square matrices of a given size (like all
2 by 2 matrices), and the set of real polynomials (or complex polynomials, or
polynomials with coefficients in any other ring) also meet the definition of a ring,
under appropriate definitions of + and ×. All rings share these properties, and
so integers and polynomials share a lot of the same properties.
– You mentioned closure under addition and multiplication, but what about sub-
traction?
Recall that you can define subtraction by a number as the addition of its additive
inverse. Since all rings have additive inverses, we can define subtraction the same
way in all of them: a − b means a + (−b). So, being closed under addition and
additive inverses implies being closed under subtraction.
– Why don’t rings have multiplicative inverses?
This question is perhaps misleading. In some rings, each nonzero element does
indeed have a multiplicative inverse, but such a ring is then called a field. The
rational numbers, real numbers, and complex numbers are fields. The integers,
however, are not. See Sample Problems.
– How do you add, subtract, and multiply polynomials?
Polynomials are added and subtracted by like terms. So,

(x2 + 4x − 3) + (2x2 + 9x + 4) = 3x2 + 13x + 1, and


(x2 + 4x − 3) − (2x2 + 9x + 4) = −x2 − 5x − 7.

Multiplication is done using the distributive property and rules of exponents.

(x2 + 4x − 3)(2x2 + 9x + 4) = x2 (2x2 + 9x + 4) + 4x(2x2 + 9x + 4) − 3(2x2 + 9x + 4)


= 2x4 + 9x3 + 4x2 + 8x3 + 36x2 + 16x − 6x2 − 27x − 12
= 2x4 + 17x3 + 34x2 − 11x − 12

– Sample Problems
(a) Give an example of a ring that is not a field.
(b) Why are the integers not a field?
(c) Why are the natural numbers not a ring?
(d) Why is the set of all quadratic polynomials not a ring?
(e) Perform the following polynomial operations:
26 CCSS for High School Math: Algebra - by Christopher Goff, Ph.D.

i. (x2 + 4x − 3) + (2x − 5)
ii. (x2 + 4x − 3) − (2x − 5)
iii. (x2 + 4x − 3)(2x − 5)
iv. (2x − 5) − (x2 + 4x − 3)

– Answers to Sample Problems


(a) Give an example of a ring that is not a field. Examples include: the integers,
polynomials, and the set of all square matrices of a given size.
(b) Why are the integers not a field? Because not every integer has a multiplica-
tive inverse that is an integer. For instance, the multiplicative inverse of 3 is
1
3
, which is not an integer.
(c) Why are the natural numbers not a ring? Because they do not contain an
additive identity (although you might see a textbook which includes 0 in the
natural numbers). Also, the additive inverses of natural numbers are not
natural numbers. For instance, the additive inverse of 3 is −3, which is not a
natural number.
(d) Why is the set of all quadratic polynomials not a ring? The set of quadratic
polynomials is not closed under multiplication. For instance, if you multiply
x2 + 1 by x2 − 1, you obtain x4 − 1, which is not quadratic.
(e) Perform the following polynomial operations:
i. (x2 + 4x − 3) + (2x − 5) = x2 + 6x − 8
ii. (x2 + 4x − 3) − (2x − 5) = x2 + 2x + 2
iii. (x2 + 4x − 3)(2x − 5) = 2x3 + 3x2 − 26x + 15
iv. (2x − 5) − (x2 + 4x − 3) = −x2 − 2x − 2

• Understand the relationship between zeros and factors of polynomials

2. Know and apply the Remainder Theorem: For a polynomial p(x) and a number a, the
remainder on division by x − a is p(a), so p(a) = 0 if and only if (x − a) is a factor of
p(x).

– What is the Remainder Theorem and how is it related to the Factor Theorem?
The way that the Remainder Theorem is stated in the Common Core standard, it
encompasses what is sometimes referred to as two different theorems, the Remain-
der Theorem and the Factor Theorem. You may recall that the Factor Theorem
What Every Math Teacher Should Know 27

says that (x − a) is a factor of p(x) if and only if p(a) = 0. This is the second
part of the Common Core version of the Remainder Theorem.
The first part of the Common Core version of the Remainder Theorem says that
if p(x) is divided by (x − a), then the remainder is p(a). To see this, recall that if
you divide p(x) by (x − a), you get a quotient polynomial q(x) and a remainder
polynomial r(x) with the degree of r(x) smaller than the degree of (x − a). (For
a refresher of division, see # 6 below, page 40.) So r(x) must be a constant here,
say r, since (x − a) has degree 1. Hence we have

p(x) = (x − a)q(x) + r.

Letting x = a gives the Remainder Theorem: p(a) = 0q(a) + r = r. Therefore,


we also obtain the special case called the Factor Theorem: p(a) = 0 if and only if
the remainder is zero, i.e., exactly when (x − a) is a factor of p(x).
3. Identify zeros of polynomials when suitable factorizations are available, and use the zeros
to construct a rough graph of the function defined by the polynomial.
– What are some ways to factor a polynomial?
In light of the Remainder Theorem (and its special case called the Factor The-
orem), we can find factors (x − a) by evaluating the polynomial at a (that is,
finding p(a)) and seeing when we obtain a zero. This can be done directly, or
via a method called “synthetic substitution” which is equivalent to doing a long
division problem with (x − a) as the divisor.
So, if we want to divide a polynomial p(x) by a linear term of the form x − a,
then by performing synthetic substitution, we not only determine the remainder
p(a), but we also determine the quotient q(x). That way, if x − a turns out to be
a factor of p(x), then we will have already factored p(x) as (x − a)q(x).
There is also a way to factor quadratic polynomials like x2 + bx + c, where b and
c are integers. It involves finding two factors of c that add up to −b. To see why
this is the case, consider a polynomial already in factored form: (x − s)(x − t). If
we multiply these factors together, we can find the original polynomial: x2 − sx −
tx + st = x2 − (s + t)x + st. So if you have a polynomial of the form x2 + bx + c,
and if you can find factors s and t of c that add up to −b, then your polynomial
will factor as (x − s)(x − t).
For example, consider x2 − 5x − 6. We need factors of −6 that add up to 5. It
seems that +6 and −1 are the desired factors. So, x2 − 5x − 6 = (x − 6)(x + 1).
28 CCSS for High School Math: Algebra - by Christopher Goff, Ph.D.

There are of course many other ways to factor quadratics (and polynomials in gen-
eral), including ones with a leading term that is not equal to 1 (such as grouping,
the “box” method, factoring tiles, etc.). Please refer to a good algebra textbook
for more information.

– How do you sketch a graph of a polynomial if you know its factors?


For more information on Graphing, see #10-11 below, starting on page 86.
If x − a is a factor of a polynomial p(x), then the value a is called a zero of p(x)
because p(a) = 0. The value a is also called a “root” of p(x). Since the graph is
the set of points where y = p(x), the graph must pass through the point (a, 0) on
the x-axis. The same is true for all the other factors of the form x − a. Hence the
zeros of p(x) are exactly the same as its x-intercepts.
I also usually find the y-intercept of any polynomial I graph, especially if I start
with a formula for p(x) because then the y-intercept is just the constant term.
You can also find it by evaluating p(0).
Putting all of these points on the graph, combined with a basic knowledge of
polynomial behavior, leads to a good qualitative picture of the polynomial.

– What does “polynomial behavior” mean? How is it relevant to graphing polyno-


mials?
First, since polynomials are defined using only addition (and subtraction) and
multiplication of numbers and variables, they are defined everywhere; the domain
of a real polynomial is the set of real numbers. This means there are no holes or
vertical asymptotes in the graph. The graph of a polynomial is also a continuous
curve. This is often translated in graphing terms by saying that you can graph
polynomials without lifting your pencil.
Second, the graph intersects the x-axis at each of its zeros. If a zero has mul-
tiplicity 1, then the graph simply crosses the x-axis at that zero. If a zero has
multiplicity 2, then the graph is tangent to the x-axis at that zero, but doesn’t
cross it. More generally, the graph is tangent to the x-axis without crossing it if
and only if there is a root at that point with even multiplicity – similar to what
happens on the basic y = x2 graph at the origin. In contrast, triple roots (and
other higher odd-multiplicity roots) will occur at points where the graph crosses
the x-axis, but it will flatten out while doing so, to the point of having a horizon-
tal tangent line at the root itself – similar to what happens on the basic y = x3
What Every Math Teacher Should Know 29

graph at the origin.


Third, the end behavior (i.e., what happens as x goes to ±∞) of a polynomial
plays a role in its graph. Let’s discuss what happens to the graph of a polynomial
p(x) as x → ∞ and let’s assume that the degree of p(x) is not zero (i.e., p(x)
is not a constant function). Since the highest power of x is positive and has a
fixed coefficient, as x increases without bound, there will come a point where the
leading term of p(x) will dominate the behavior. As x → ∞, xn → ∞ as well. So,
as you go to the right, every basic polynomial graph will rise without bound. Of
course, the sign of the leading term will therefore determine if we have a positive
or negative end behavior as x → ∞.
The story as x → −∞ is similar, in that the leading term still dominates the
behavior, but different in that now the behavior depends on the highest exponent.
If p(x) has degree n and n is even, then as x → −∞, xn → ∞. But if n is odd,
then as x → −∞, xn → −∞. As before, if the leading coefficient is positive, then
p(x) will have the same behavior as xn , but if the leading coefficient is negative,
then the signs on the end behavior will change.
Another way to phrase this is: for polynomials of even degree (at least 2), if
the leading coefficient is positive (respectively, negative), then the end behavior
as x → ±∞ is positive (resp., negative). For polynomials of odd degree, if the
leading coefficient is positive (resp., negative), then as x → ∞, the end behavior is
positive (resp., negative), while as x → −∞, the end behavior is negative (resp.,
positive).
– Can we do an example?
Let’s find the roots of p(x) = x5 + 8x4 + 19x3 + 8x2 − 20x − 16 and sketch a graph.
Using the Remainder Theorem (and synthetic substitution), we notice that both
1 and −1 are roots, which shortens the calculations:
x 1 8 19 8 −20 −16
1 1 9 28 36 16 0 root
−1 1 8 20 16 0 root
−1 1 7 13 3 (not a double root)
−2 1 6 8 0 root
−2 1 4 0 double root
−4 1 0 root
(You can also try plugging in 1, −1, −2, and −4 into the polynomial to obtain
30 CCSS for High School Math: Algebra - by Christopher Goff, Ph.D.

zero. Review synthetic substitution if you wish to use it.) The roots are : 1, −1,
−2, and −4, where −2 is a double root. This also means that

p(x) = x5 + 8x4 + 19x3 + 8x2 − 20x − 16 = (x − 1)(x + 1)(x + 2)2 (x + 4).

So, the graph y = p(x) has four x-intercepts (at 1, −1, −2, and −4, with the
root at −2 being a double root) and one y-intercept (at −16). The polynomial
p(x) has degree 5 and a positive leading coefficient (of 1), and thus as x → ±∞,
p(x) → ±∞ (i.e., + goes with + and − with −). So, plotting the intercepts,
remembering the continuity, knowing the end behavior, and knowing the behavior
at a double root, we can sketch the following graph of y = p(x).

There are some Sample Problems involving polynomial zeros and graphing located
at the end of #5, below.

• Use polynomial identities to solve problems

4. Prove polynomial identities and use them to describe numerical relationships. For example,
the polynomial identity (x2 + y 2 )2 = (x2 − y 2 )2 + (2xy)2 can be used to generate
Pythagorean triples.
What Every Math Teacher Should Know 31

– What is a polynomial identity?


A polynomial identity is an equation involving polynomial expressions that is
always true, regardless of the particular values the variables might have. A sim-
ple (though not particularly useful) example is: x = x. This is certainly true,
regardless of what value x may have. Another example is:

(x − y)(x + y) = x2 − y 2 .

This is an identity, because it is true for every choice you can make for the values
of x and y. There are of course many different polynomial identities.
– How do you prove or disprove a polynomial identity?
The usual way to prove a polynomial identity is to manipulate the expression on
one side of the equation using reversible rules of algebra until it looks exactly
like the expression on the other side of the equation. In our example above, we
can multiply out the factors on the left hand side and then gather like terms to
simplify.
(x − y)(x + y) = x2 + xy − xy − y 2 = x2 − y 2

Notice that we never had to evaluate any of the variables at a number, but we
were able to make the two sides identical.
You can disprove a polynomial identity by just finding a value of one or more
variables that makes the equation false. For example, if someone were to claim
that x2 + y 2 = (x − y)(x + y) is an identity, you could disprove the claim by letting
x = y = 1, for example, for then the equation becomes

12 + 12 = (1 − 1)(1 + 1) or 2 = 0,

which is clearly false.


– What do you mean about “reversible” rules of algebra?
In algebra, there are things that you can do to an expression that can be undone
in such a way that you wind up where you started. For example, adding 3 to
an expression can be undone by subtracting 3 (or adding −3). Multiplying by 3
can be undone by dividing by 3 (or multiplying by 31 ). These kinds of steps are
reversible.
But sometimes you can make a step that isn’t totally reversible. For instance,
squaring an expression gives you a new expression, but taking the square root
32 CCSS for High School Math: Algebra - by Christopher Goff, Ph.D.

of your answer might not take you back to where you started. For example, if
x = −3, and we form x2 , then the answer is 9. Trying to get back to x either leads

us to 3 (when one just finds 9 without worrying about the sign), or it takes us
to 3 or −3 (also known as ±3). So squaring an expression is not a reversible step.
We have to avoid steps like that when we are trying to prove identities.

As an example, try to prove that x2 − 6x + 9 = x − 3. If we square both sides,
we get x2 − 6x + 9 = x2 − 6x + 9, which is clearly a polynomial identity, even
though the expression we started with on the left hand side was technically not
a polynomial expression because of the square root. But the original equation is
not true if x < 3. You can even check a number like x = 0. The equation becomes

9 = −3, which is false.

– How can you use their example to generate Pythagorean triples?


This is actually quite a famous example from the history of mathematics. You
can find formulas very close to these in Euclid’s Elements (Book X, Proposition
29), written in about 300 BC. First, we will prove that it really is a polynomial
identity, and then we will examine its structure more closely.
Working with the right hand side, we get

(x2 − y 2 )2 + (2xy)2 = x4 − 2x2 y 2 + y 4 + 4x2 y 2


= x4 + 2x2 y 2 + y 4
= (x2 + y 2 )2 .

So, the equation is truly an identity.


Now, if we were to make a substitution (in order to see the structure better), by
letting c = x2 +y 2 , a = x2 −y 2 , and b = 2xy, then our identity becomes c2 = a2 +b2 ,
which has the same structure as the Pythagorean Theorem in that two squares
add together to equal a third. Because we are working with an identity, we have
a true statement regardless of the values chosen for x and y. Therefore, if you
want to create a Pythagorean triple, you can just select values for x and y, and
plug them in to determine a, b, and c.
For example, let’s choose x = 2 and y = 1. Then a = x2 − y 2 = 4 − 1 = 3,
b = 2xy = 2(2)(1) = 4, and c = x2 + y 2 = 4 + 1 = 5. So we get a 3-4-5 right
triangle in this case. For another example, let’s choose x = 5 and y = 4. Then
What Every Math Teacher Should Know 33

a = 9, b = 40, and c = 41. And yes,

92 + 402 = 81 + 1600 = 1681 = 412 .

Notice that to avoid negative numbers, we picked x > y, but you would still get a
Pythagorean triple, even if x ≤ y. See the Sample Problems for more about this
and other cases.
– What is another example of how to use polynomial identities?
There are a number of possibilities that depend upon the specific situation. But
let’s consider the following equation:

(2x)2 + (2y + 1)2 = 2(2x2 + 2y 2 + 2y) + 1

First, let’s show it’s an identity, and then let’s interpret it. Starting with the
left-hand side,

(2x)2 + (2y + 1)2 = 4x2 + 4y 2 + 4y + 1


= 2(2x2 + 2y 2 + 2y) + 1

So, yes, it’s an identity. But what could it mean?


One interpretation has to do with numbers being even or odd. An even number
is one that can be written as 2x for some integer value of x. The converse is true
too. If x is any integer, then 2x is even. Similarly, if y is an integer, then 2y + 1
is odd. Moreover, any odd number can be written as 2y + 1 for some integer y.
So, it looks like our identity is saying that if you square an even number and an
odd number, and you add them together, you have to get an odd number. That’s
because the number on the right hand side is one more than twice an integer.
But how do we know that the quantity in parentheses 2x2 + 2y 2 + 2y is an in-
teger? Well, since this interpretation requires x and y to be integers, and since
polynomials only involve adding and multiplying, we know that we must get an
integer outcome for 2x2 + 2y 2 + 2y. Therefore, the square of an even number plus
the square of an odd number is odd.
5. (+) Know and apply the Binomial Theorem for the expansion of (x + y)n in powers of x
and y for a positive integer n, where x and y are any numbers, with coefficients determined
for example by Pascal’s Triangle.5
5
The Binomial Theorem can be proved by mathematical induction or by a combinatorial argument.
34 CCSS for High School Math: Algebra - by Christopher Goff, Ph.D.

– What is the Binomial Theorem?


The Binomial Theorem gives a way to calculate the coefficients of various terms
in a power of a binomial. Specifically,
n
!
X n
(x + y)n = xn−k y k ,
k=0 k
!
n n!
where = and is read “n choose k.” It is also the number of ways
k k!(n − k)!
to choose k objects from a set of n objects. Recall that by convention, 0! = 1.
The Binomial Theorem can be proved by mathematical induction.
Proof: We start by checking that the formula is true for n = 1.
1
! ! !
X 1 1 1
x1−k y k = x1 y 0 + x0 y 1 = 1x + 1y = (x + y)1 .
k=0 k 0 1
Now we show that whenever the formula is true for some value of n then it is also
true for n + 1. (Via induction, this will imply that the formula is true for any
value of n.)

(x + y)n+1 = (x + y)(x + y)n


n
! !
X n
= (x + y) xn−k y k
k=0 k
n
! n
!
X n X n
= xn−k+1 y k + xn−k y k+1
k=0 k k=0 k
We need to re-index the second summation in order to combine like terms cor-
rectly. Let ` = k + 1 so that the summation is from ` = 1 to ` = n + 1. Then
n
! n+1
!
X n X n
(x + y)n+1 = xn−k+1 y k + xn−(`−1) y `
k=0 k `=1 ` − 1
Notice that we can combine the middle terms (1 ≤ k, ` ≤ n) and notice that we
have like terms now, if we match up k in the first sum with ` in the second, but
that two terms need to be separated out, namely the k = 0 term and the ` = n+1
term.
n
" ! !# !
X n n
(x + y)n+1 = xn+1 + + xn−k+1 y k + y n+1
k=1 k k−1
n+1
!
X n+1
= xn+1−k y k ,
k=0 k
What Every Math Teacher Should Know 35

which is exactly what we wanted to show. !2 ! !


n n n+1
Notice that here we used the fact that + = , which
k k−1 k
can be proved directly (See Sample Problems.), but will also be reflected in how
to find the numbers in Pascal’s Triangle.
Example: Expand (x + 2)5 .

5
!
X 5
(x + 2)5 = x5−k 2k
k=0 k
! ! !
5 5 5
= x5 20 + x4 21 + . . . + x0 25
0 1 5
= x5 + 5x4 (2) + 10x3 (4) + 10x2 (8) + 5x(16) + 1(32)
= x5 + 10x4 + 40x3 + 80x2 + 80x + 32.

– What is Pascal’s Triangle? How can you use it to obtain binomial coefficients?
Studied at length by Blaise Pascal6 in the mid-17th century, what is now called
Pascal’s Triangle carries a lot of information. We will build it inductively below,
but point
! out that the numbers we obtain will be values of the “choose” function
n
described above.
k
We start with the number 1 at the top left vertex of our triangle.7 Next we will
write a 1 to the right of it and below it.

1 1
1 =⇒ =⇒ . . .
1

Now we’ll add a third diagonal row, starting and ending with 1, but whose middle
number is determined by adding the number above it to the number to its left
(both underlined), which gives us a 2 (bold face) in this case. Then we’ll go to a
fourth row, again starting and ending with 1, but determining the inner numbers
by adding the upper and left neighbors together. (Here, we have underlined the

6
Though named for Pascal, this triangle of numbers was known to many cultures (Greece, China, etc.) centuries
before Pascal.
7
I’ll draw it the way Pascal did, with a diagonal bottom, because it’s easier for me to do so. An internet search
on Pascal’s Triangle will turn up several images of the more typical “flat-bottomed” approach.
36 CCSS for High School Math: Algebra - by Christopher Goff, Ph.D.

1 and 2 that add together to make the 3 in bold face.)

1 1 1 1
1 1 1
1 2 3
. . . =⇒ 1 2 =⇒ =⇒ . . .
1 3
1
1

We’ll write out two more steps in the pattern.

1 1 1 1 1 1
1 1 1 1 1
1 2 3 4 5
1 2 3 4
1 3 6 10
. . . =⇒ 1 3 6 =⇒ =⇒ . . .
1 4 10
1 4
1 5
1
1

This pattern continues for as long as you wish.


The numbers that are appearing have combinatorial meanings. If we start by
numbering the rows and columns from row zero at the top and go downward (to
row 1, row 2, etc.), and column zero at the left and go to the right (to column 1,
column 2, etc.), then the entry in row i and column j is either
! !
i+j i+j
or ,
i j

both of which are equal. So if you want to know how many ways there are to
choose a committee of three people from a group of five people, then you can go
to row 3 and column 2 (or row 2, column 3), which gives 10. (Remember, the
first row is row zero, and the first column is column zero.) There are ten ways to
select a committee of three from a group of five. Notice that this agrees with the
formula above: !
5 5! 5·4·3·2·1
= = = 10.
3 3! · 2! (3 · 2 · 1)(2 · 1)
– What did you mean earlier when you said that a certain combinatorial identity
is reflected in how to find numbers in Pascal’s Triangle?
The way that numbers in the triangle are found involves adding two other ele-
ments in the triangle together. If each entry corresponds to a value of the choose
What Every Math Teacher Should Know 37

function, then we get an identity involving values of the choose function. Let’s
check what that means for the entry in row k and column n + 1 − k. It will equal
the sum of the entry in row k and column n − k and the entry in row k − 1 and
column n + 1 − k. In terms of the choose function, we get
! ! !
(n + 1 − k) + k (n − k) + k (n + 1 − k) + (k − 1)
= + ,
k k k−1

which simplifies to
! ! !
n+1 n n
= + .
k k k−1

This is exactly what we needed in order to prove the Binomial Theorem. You
could think of Pascal’s Triangle as a picture of the choose function, and the combi-
natorial identity provides the addition rule for finding the next row in the triangle.
Notice we haven’t yet proved the identity. That’s for the Sample Problems.
– Sample Problems
(a) Factor the polynomial x4 − 5x2 + 4 and sketch a quick graph.
(b) Use the Remainder Theorem (and synthetic substitution or long division) to
factor 2x3 − x2 − 13x − 6. Then sketch a quick graph.
(c) Recall the identity: (x2 − y 2 )2 + (2xy)2 = (x2 + y 2 )2 . What happens if x < y?
Start with x = 1, y = 2. Then try x = 4, y = 5. What do you notice? [Hint:
compare to the other two examples given in the text.]
(d) Now try the identity mentioned in the previous problem with x = −2, y = 1.
Do you get a “Pythagorean Triple” still? Why or why not? What’s going on
here?
(e) Expand (x − 3)4 using the Binomial Theorem.
(f) Expand (2a + 5b)3 using the Binomial Theorem.
! ! !
n+1 n n
(g) Prove = + .
k k k−1
– Answers to Sample Problems
(a) Factor the polynomial x4 − 5x2 + 4 and sketch a quick graph.

x4 − 5x2 + 4 = (x2 − 4)(x2 − 1) = (x + 2)(x − 2)(x + 1)(x − 1)


38 CCSS for High School Math: Algebra - by Christopher Goff, Ph.D.

(b) Use the Remainder Theorem (and synthetic substitution or long division) to
factor 2x3 − x2 − 13x − 6. Then sketch a quick graph.
x 2 −1 −13 −6
1 2 1 −12 −18
2 2 3 −7 −20
3 2 5 2 0 root
−1 2 3 −1
−2 2 1 0 root
− 12 2 0 root
So the roots are 3, −2, and − 21 .
What Every Math Teacher Should Know 39

(c) Recall the identity: (x2 − y 2 )2 + (2xy)2 = (x2 + y 2 )2 . What happens if x < y?
Start with x = 1, y = 2. Then try x = 4, y = 5. What do you notice? [Hint:
compare to the other two examples given in the text.]
Let x = 1 and y = 2. Then a = x2 − y 2 = −3, b = 2xy = 4, and c = x2 + y 2 =
5. We get (−3, 4, 5), which is technically not a Pythagorean triple because of
the negative number, but it does still satisfy the equation: a2 + b2 = c2 .
If x = 4 and y = 5, then we get a = −9, b = 40, and c = 41, which is still a
solution to a2 + b2 = c2 , but could not be the sides of a right triangle because
of the negative sign.
If we look more closely at the formulas, you see that switching x and y will
change the sign on a, while b and c remain unchanged. So if 0 < x < y, we
get a negative value for a, but positive for b and c. If we were to switch x
and y then we would obtain a true Pythagorean triple in the sense that all
the numbers would be positive.
(d) Now try the identity mentioned in the previous problem with x = −2, y = 1.
Do you get a “Pythagorean Triple” still? Why or why not? What’s going on
here?
Let x = −2 and y = 1. Then a = 3, b = −4, and c = 5. So now the sign of b
has changed from what it was. Looking at the formulas for a, b, and c, we see
that if we change the sign on x, then only the sign of b changes. So while we
would technically not get a possible set of values for sides of a right triangle,
we would still get a solution to the equation a2 + b2 = c2 .
For fun, convince yourself that if xy < 0 (i.e.. exactly one of them is negative),
and if |x| < |y|, then both a and b are negative, and yet still a2 + b2 = c2 .
(e) Expand (x − 3)4 using the Binomial Theorem.
Here, we let y = −3, and n = 4 in the general formula.

(x − 3)4 = x4 + 4x3 (−3) + 6x2 (−3)2 + 4x(−3)3 + (−3)4


= x4 − 12x3 + 54x2 − 108x + 81.

(f) Expand (2a + 5b)3 using the Binomial Theorem.


Here, we let x = 2a, y = 5b, and n = 3.

(2a + 5b)3 = (2a)3 + 3(2a)2 (5b) + 3(2a)(5b)2 + (5b)3


= 8a3 + 60a2 b + 150ab2 + 125b3 .
40 CCSS for High School Math: Algebra - by Christopher Goff, Ph.D.

! ! !
n+1 n n
(g) Prove = + .
k k k−1
We will write the right-hand side using the factorial definition of the choose
function and then combine terms by obtaining a common denominator.
! !
n n n! n!
+ = +
k k−1 k!(n − k)! (k − 1)!(n − k + 1)!
n!(n − k + 1) n!k
= +
k!(n − k)!(n − k + 1) (k − 1)!(n − k + 1)!k
n!(n + 1 − k) n!k
= +
k!(n − k + 1)! k!(n − k + 1)!
n!(n + 1 − k) + n!k
=
k!(n + 1 − k)!
n!(n + 1) − n!k + n!k
=
k!(n + 1 − k)!
n!(n + 1)
=
k!(n + 1 − k)!
!
(n + 1)! n+1
= =
k!(n + 1 − k)! k

• Rewrite rational expressions


6. Rewrite simple rational expressions in different forms; write a(x)/b(x) in the form q(x) +
r(x)/b(x), where a(x), b(x), q(x), and r(x) are polynomials with the degree of r(x)
less than the degree of b(x), using inspection, long division, or, for the more complicated
examples, a computer algebra system.
– What is division in the natural numbers (or integers)?
There is a Division Algorithm in the integers that says the following. If a and b
are natural numbers, then there exist unique integers q and r (called the quotient
and remainder) satisfying
i. a = qb + r, and
ii. 0 ≤ r < b.
Usually, a ≥ b, although that is not technically necessary. Also, a could be any
integer and division would still work. Another way of writing this equation is to
divide both sides by b, which gives
a r
=q+ ,
b b
What Every Math Teacher Should Know 41

r
where the fraction 0 ≤ b
< 1.
– How does this relate to polynomials?
We saw beginning on page 24 that the polynomials form a system that is similar
to the integers in that polynomials are closed under addition, subtraction, and
multiplication. Polynomials also have a notion of size (the degree of the polyno-
mial) that is similar to the absolute value of an integer. So it turns out that you
can divide polynomials in much the same way you can divide integers.
Division in polynomials can be stated as: If a(x) and b(x) are polynomials, and
b(x) is not identically zero, then there exist unique polynomials q(x) and r(x)
satisfying
i. a(x) = q(x)b(x) + r(x), and
ii. the degree of r(x) is less than the degree of b(x).
Note that the degree of a polynomial has to be greater than or equal to zero, and
so we don’t need to state that explicitly above. So, if we divide both sides by the
polynomial b(x), which we can do because it is not identically zero, then we get
a(x) r(x)
= q(x) + ,
b(x) b(x)
and the degree of r(x) is less than the degree of b(x).
– What are some examples of division using inspection?
Let’s say you want to divide x2 +7x+6 by x+1. You might notice that x2 +7x+6
can be factored as (x + 1)(x + 6), and so
x2 + 7x + 6
= x + 6.
x+1
Along those lines, though, one can get “creative.” You could divide x2 + 7x by
x+1 by adding and subtracting 6 before you begin, and then using rules of algebra
to simplify the answer:
x2 + 7x x2 + 7x + 6 − 6 x2 + 7x + 6 6 6
= = − = (x + 6) − .
x+1 x+1 x+1 x+1 x+1
So you can see that q(x) = x + 6 and r(x) = −6 in this case. Note that the degree
of r(x) is zero, which is less than 1, the degree of the divisor b(x) = x + 1.
Let’s do one more example using this idea of rewriting the numerator by adding
terms. Divide x2 + 1 by x2 − 1.
x2 + 1 x2 − 1 + 2 x2 − 1 2 2
2
= 2
= 2
+ 2 =1+ 2 .
x −1 x −1 x −1 x −1 x −1
42 CCSS for High School Math: Algebra - by Christopher Goff, Ph.D.

The quotient polynomial is the constant 1 and the remainder polynomial is the
constant 2.
– What is an example of division using long division?
Let’s divide x4 − 3x3 + x − 1 by x2 + x + 1 and see what we get. (You might want
to review long division of polynomials.)

x2 − 2x − 4
x2 − x + 2 |x4 − 3x3 + 0x2 + x − 1
x4 − x3 + 2x2
−2x3 − 2x2 + x
−2x3 + 2x2 − 4x
−4x2 + 5x − 1
−4x2 + 4x − 8
x+7

So the quotient is q(x) = x2 − 2x − 4 and the remainder is r(x) = x + 7.


In the case where the divisor b(x) is a linear polynomial in the form x − c, then
you know that the remainder must be a constant. The Remainder Theorem tells
you that the remainder is a(c). You can do “synthetic substitution” to determine
the quotient and the remainder in this case. (Review it if you wish to use it.) For
example, let’s divide x3 + 5x2 − 2x − 10 by x + 2. Using synthetic substitution,
we obtain:
x 1 5 −2 −10
−2 1 3 −8 6
This means that the quotient polynomial is x2 + 3x − 8 and that the reminder
polynomial (underlined) is the constant 6.
– What is an example of division using a computer algebra system?
Using a computer algebra system goes beyond the scope of this book. If you
have trouble getting it to work, consult the manual or help menu for whichever
software you are using.

7. (+) Understand that rational expressions form a system analogous to the rational num-
bers, closed under addition, subtraction, multiplication, and division by a nonzero rational
expression; add, subtract, multiply, and divide rational expressions.
What Every Math Teacher Should Know 43

– How can rational expressions be analogous to rational numbers?


What this means is that from an algebraic viewpoint (i.e., from the properties they
have), rational expressions and rational numbers have a lot in common. Indeed
they are both examples of fields (mentioned earlier), i.e., rings with multiplicative
inverses.
Let Q be the set of rational numbers (that is, the set of numbers that can be
a
written with a and b integers, and b 6= 0) and let F be the set of rational
b
,
p(x)
expressions , where p(x) and q(x) are any polynomials with real coefficients
q(x)
and q(x) 6= 0. We will explicitly show that Q shares some key properties with F .
(a) Both Q and F each contain a multiplicative identity element.
1
The element 1 = 1
is an element of both Q and F , because 1 is an integer
and a (degree zero) polynomial.
(b) Both Q and F are closed under multiplication.

a c ac
· =
b d bd
Since b and d are not zero, bd 6= 0. Since integers are closed under multipli-
ac
cation, ac and bd are both integers. Hence bd
is in Q. Similarly,
p(x) a(x) p(x)a(x)
· =
q(x) b(x) q(x)b(x)
Polynomials are closed under multiplication. Since q(x) and b(x) are not 0,
then q(x)b(x) 6= 0. So the product of two elements of F is another element of
F.
(c) Both Q and F are closed under addition.

a c ad + bc
+ =
b d bd
Again, since b and d are not zero, neither is bd. The numerator and denom-
inator must be integers because integers are closed under multiplication and
ad+bc
addition, and so bd
is in Q. Similarly,
p(x) a(x) p(x)b(x) + q(x)a(x)
+ =
q(x) b(x) q(x)b(x)
Again, since polynomials are closed under multiplication and addition, and
since the denominator is not zero, the sum of two elements of F is again an
element of F .
44 CCSS for High School Math: Algebra - by Christopher Goff, Ph.D.

(d) Every non-zero element of Q and of F is invertible.


a b
If b
6= 0, then a 6= 0. So therefore a
is an element of Q. It’s clear that
a p(x)
b
· ab = 1, and so b
a
is the inverse of ab . Similarly, if q(x)
6= 0, then p(x) 6= 0.
q(x)
So that means that p(x)
is an element of F . Multiplying, we get

p(x) q(x) p(x)q(x)


· = = 1.
q(x) p(x) p(x)q(x)
Hence, every non-zero element of Q and of F is invertible.
(e) The sets Q and F are closed under division by a non-zero element.
Since we just saw that each non-zero element is invertible, we can therefore
divide by it. (After all, division by an element is just multiplication by its
inverse.)
– What are some examples?
2x + 4 x2 − 5x + 6
Consider f (x) = and g(x) = . We will add, subtract, multiply,
x−3 x+2
and divide these expressions.
(a) f (x) + g(x). We first need to get a common denominator so that we can
combine the terms. In this case, the least common denominator is the product
(x − 3)(x + 2)
2x + 4 x2 − 5x + 6 (2x + 4)(x + 2) (x2 − 5x + 6)(x − 3)
+ = +
x−3 x+2 (x − 3)(x + 2) (x + 2)(x − 3)
2
2x + 8x + 8 x − 8x2 + 21x − 18
3
= +
(x + 2)(x − 3) (x + 2)(x − 3)
3 2
x − 6x + 29x − 10
= .
(x + 2)(x − 3)
We can try to factor the numerator, but since neither (x + 2) nor (x − 3) is
a factor of the numerator (which you can verify by long division or synthetic
substitution), then we might as well leave it.
(b) f (x) − g(x). We still need to get a common denominator, and a lot of the
previous calculation still applies. In fact, it’s only the last step that is different:
2x2 + 8x + 8 x3 − 8x2 + 21x − 18 −x3 + 10x2 − 13x + 26
− = .
(x + 2)(x − 3) (x + 2)(x − 3) (x + 2)(x − 3)
Make sure that you subtract each and every term of the numerator of the
fraction being subtracted. This is a very common error when subtracting
rational expressions. Again, nothing will cancel, so we stop here.
What Every Math Teacher Should Know 45

(c) f (x)g(x). We do not need a common denominator here, but we can try to
factor each expression, and then cancel common factors if possible. Remember
that only common factors can be canceled. Lots of errors are made when
students try to cancel common terms between a numerator and denominator,
which is not algebraically valid.

2x + 4 x2 − 5x + 6 2(x + 2) (x − 3)(x − 2)
· = ·
x−3 x+2 x−3 x+2
= 2(x − 2) = 2x − 4.

(d) f (x)/g(x). Here, we will think of division as multiplication by the reciprocal.


1
So we will really perform f (x) · g(x)
.

2x + 4 x+2 2(x + 2) x+2


· 2 = ·
x − 3 x − 5x + 6 x−3 (x − 3)(x − 2)
2(x + 2)2
= .
(x − 3)2 (x − 2)

– Sample Problems
(a) Rewrite the numerators creatively in order to find the quotient and remainder
polynomials.
x2
i. 2
x +2
x3 + x2 − 1
ii. 2
x +x+1
(b) Use long division to divide x4 − 6x3 − 10x2 − 24x + 30 by x2 − 2x + 4.
x2 − 2x + 1 x2 + 5x
(c) Let f (x) = and let g(x) = .
x+5 x−3
i. Find f (x) + g(x).
ii. Find f (x) − g(x).
iii. Find f (x) · g(x).
f (x)
iv. Find .
g(x)
– Answers to Sample Problems
(a) Rewrite the numerators creatively in order to find the quotient and remainder
polynomials.
x2 x2 + 2 − 2 2
i. 2 = 2
=1− 2 . So q(x) = 1 and r(x) = −2.
x +2 x +2 x +2
46 CCSS for High School Math: Algebra - by Christopher Goff, Ph.D.

x3 + x2 − 1 x3 + x 2 + x − x − 1 x3 + x2 + x x+1
ii. 2
= 2
= 2
− 2 =
x +x+1 x +x+1 x +x+1 x +x+1
x+1
=x− 2 . So q(x) = x and r(x) = −x − 1.
x +x+1
(b) Use long division to divide x4 − 6x3 − 10x2 − 24x + 30 by x2 − 2x + 4.

x2 − 4x − 22
x2 − 2x + 4 |x4 − 6x3 − 10x2 − 24x + 30
x4 − 2x3 + 4x2
−4x3 − 14x2 − 24x
−4x3 + 8x2 − 16x
−22x2 − 8x + 30
−22x2 + 44x − 88
−52x + 118

So, q(x) = x2 − 4x − 22 and r(x) = −52x + 118.


x2 − 2x + 1 x2 + 5x
(c) Let f (x) = and let g(x) = .
x+5 x−3
2x3 + 5x2 + 32x − 3
i. Find f (x) + g(x).
(x + 5)(x − 3)
−15x2 − 18x − 3
ii. Find f (x) − g(x).
(x + 5)(x − 3)
x(x − 1)2
iii. Find f (x) · g(x).
x−3
2
f (x) (x − 1) (x − 3)
iv. Find .
g(x) x(x + 5)2
What Every Math Teacher Should Know 47

CREATING EQUATIONS**

• Create equations that describe numbers or relationships


1. Create equations and inequalities in one variable and use them to solve problems. In-
clude equations arising from linear and quadratic functions, and simple rational and
exponential functions.
– How do you create equations and inequalities?
If expressions are the nouns in the language of algebra, then equations and in-
equalities are the sentences. They give us some information about the objects
we are studying. So if we have information about some numbers, then we can
translate that information into an equation or inequality.
As an example, suppose that you bought four children’s movie tickets and the
cost was $38. We can represent this as an equation. If we let P represent the
price of a child’s movie ticket, then four tickets would cost 4P . The equation is
therefore 4P = 38. Note that we are not yet solving equations - that will come
later. Right now we are only creating them.
Now suppose that you don’t know exactly what the price of the four children’s
movie tickets was, but that you did know that it was less than $40. We can
represent this as an inequality. Again, if P is the price of one child’s ticket, then
4P is the price of four children’s tickets. The inequality is therefore 4P < 40.
Again, we are not solving inequalities yet. That doesn’t happen until we get to
the section Reasoning with Equations and Inequalities.
– How do you use equations and inequalities to solve problems?
We will be working with equations and inequalities that contain variables. That
means that there is some unknown quantity in the “sentence.” When you solve an
equation or inequality, you are finding out the values of the variable(s) that will
make the sentence a true statement. So we will create an equation or inequality
in such a way that by solving it, we will get the answer we desire.
In the former example above, we created the equation 4P = 38. We could solve
this equation for P and that would give us the price for one child’s movie ticket.
In the latter example, we created the inequality 4P < 40. If we were to solve this,
then it would tell us the maximum price that a single child’s ticket could cost.
48 CCSS for High School Math: Algebra - by Christopher Goff, Ph.D.

Since the information obtained is slightly different in each case, we want to create
the equation or inequality that will lead us to the information we seek.
– What are some linear and quadratic examples?
We have already seen some simple linear examples. For another example, assume
that your cell phone contract costs $20 a month plus $0.05 per minute for calls.
We’ll ignore texting and data for now. Suppose your bill for last month was
$42.15. We can write an equation to represent this. If we let m represent the
number of minutes spent calling, then the total cost of those minutes is 0.05m,
and so the cost of your monthly bill is 20 + 0.05m. So, our equation representing
last month’s bill is:
20 + 0.05m = 42.15.

Suppose that we want the bill to be at most $40. We can write that as an
inequality: 20 + 0.05m ≤ 40.
For a quadratic example, let’s turn to economics. One simple model says that the
price of a product determines the quantity sold. Let’s suppose that the quantity
sold is 3200 − 25p, where p is the price of the item in dollars. Now suppose that
the total revenue is $32,175. We can write an equation to represent this. Since
the revenue is the total amount of money coming in, it must equal the product of
the quantity sold and the price. So, our equation is

(3200 − 25p)p = 32,175 or 3200p − 25p2 = 32,175.

Suppose that we want a revenue of at least $100,000. Then we can write that as
an inequality: 3200p − 25p2 ≥ 100,000.
– What are some rational and exponential examples?
k
The rational expression 2 can be used to model the behavior of certain physical
x
quantities, such as the force due to gravity between two bodies that are separated
by a distance, x, where k is a constant. If you wanted to determine the value(s)
2k k 2k
of x for which this force is equal to , you could create the equation 2 = .
3 x 3
If you wanted to know the values of x for which the force is greater than 2k, you
k
would create the inequality 2 > 2k.
x
For an exponential example, you may recall the expression P (1 + r)t that repre-
sents the amount of money in a bank account, where P is the initial deposit, r is
the annual interest rate, and t is the number of years that have passed since the
What Every Math Teacher Should Know 49

initial deposit. The interest is compounded annually. What does the following
equation mean?
5000(1.045)t = 7500.
This equation corresponds to P = 5000 and r = 0.045 = 4.5%. So, it says that
someone put $5000 into a bank account earning 4.5% interest, and they have
$7500 in the account, t years later. If we were to solve this equation, it would tell
us the amount of time it takes for $5000 to grow to $7500 in an account bearing
4.5% interest, compounded annually. If we wanted to know when the account had
more than $8000 dollars, then we could solve the inequality: 5000(1.045)t > 8000.
2. Create equations in two or more variables to represent relationships between quantities;
graph equations on coordinate axes with labels and scales.
– How do you create equations in two or more variables?
The process is very similar to what we have already done, but in this case there
is more than one unknown quantity in one or more of the expressions in the
equation. Let’s return to the cell phone example, where the monthly cost was
$20 a month plus $0.05 per minute for calls, but now let’s say that the plan also
costs $0.10 per text message, and that the total bill is $50.40. The equation for
this situation would thus be

20 + 0.05m + 0.1t = 50.4,

where t represents the number of text messages.


– How do you graph equations with two variables?
Graphing provides all learners (and especially visual learners) with another way
of understanding algebraic concepts better and more deeply. We will go into a
lot more detail below (See page 86.), but for now, we will graph two examples to
help remind you of how graphing works and what it means to graph.
A graph of an equation of two variables shows a visual depiction of the solutions
of that equation. Each point on the graph represents a valid solution, and each
valid solution corresponds to a point on the graph.
One of the most common graphs in algebra is of an equation in the form y =
mx + b. We will graph y = 2x + 3. We can try just plotting some points and then
seeing what happens. If x = 0, then y = 3. So we plot the point (0, 3). If x = 2
then y = 7, and so (2, 7) is also on the graph. Continuing in this way, you will
see that the points lie on a straight line, with a y-intercept of 3 and a slope of 2.
50 CCSS for High School Math: Algebra - by Christopher Goff, Ph.D.

For another example, let’s plot y = x2 − 2x. By picking x-values and calculating
the corresponding y-values, we come up with the following table.

x 0 1 2 3 4
y 0 −1 0 3 8

Plotting these points, we start to see a curved shape. You can also see some
symmetry around x = 1. For example, if x = −1, then y = 3, which is the same
y-value as when x = 3. If x = −2, we get y = 8, which also occurs when x = 4.
So, plotting all these points, we get a parabola.
What Every Math Teacher Should Know 51

3. Represent constraints by equations or inequalities, and by systems of equations and/or


inequalities, and interpret solutions as viable or nonviable options in a modeling context.
For example, represent inequalities describing nutritional and cost constraints on com-
binations of different foods.

– What is a system of equations and/or inequalities?


A system of equations and/or inequalities means that more than one equation or
inequality is happening at the same time. We may have two pieces of information
about a quantity, and we want both of those statements to be true simultaneously.
For example,
y = 2x + 3 and y = x2 + 2x + 2

makes a system of equations. The word “and” is usually not written, but we’ll
write it here because for a system of equations, one always wants to find values
for the variables that make both statements in the system true at the same time.
– How can you represent constraints by equations or inequalities?
We have already seen some examples of inequalities. A “constraint” in a modeling
problem usually means that there is some limitation (physical, monetary, nutri-
tional, etc.) that needs to be turned into an equation or an inequality, whichever
is appropriate. For example, returning to the problem of the phone bill, sup-
pose you cannot spend more than $60 per month on your phone bill. Then that
represents a budgetary constraint. The corresponding inequality would be

20 + 0.05m + 0.1t < 60.

If you have more than one constraint, then you have a system of equations and/or
inequalities. Suppose that you have to talk to your friend for at least five hours
each month. That means that you are using at least 300 minutes, i.e. m ≥ 300.
Including the previous constraint, we now have a system of inequalities:

20 + 0.05m + 0.1t < 60 and m ≥ 300.

– What does it mean for a solution to a system to be viable?


A solution to a system is viable if it makes every statement in the system true.
In the sample system above, the solution m = 500, t = 100 is viable because it
satisfies both inequalities:

20 + 0.05(500) + 0.1(100) = 20 + 25 + 10 = 55 < 60 and 500 ≥ 300.


52 CCSS for High School Math: Algebra - by Christopher Goff, Ph.D.

On the other hand, the solution m = 1000, t = 0 is not viable because even though
1000 ≥ 300, the other inequality is false for these values of m and t:

20 + 0.05(1000) + 0.1(0) = 20 + 50 + 0 = 70 6< 60.

By the way, systems of constraints do not have to be made up of inequalities.


Constraints can be equalities as well. For instance, we can check if the following
point (3, −4) is a valid solution to the system of constraints

x2 + y 2 = 25 and x + y = −1.

A quick check shows that this solution is indeed viable.


– What does their example mean about nutritional and cost constraints?
Most people want to eat better. That usually means buying more nutritious food.
However, more nutritious food often costs more as well. So there is a balance of
two constraints, a minimal amount of nutrition you want and a maximal amount
of cost you can afford to pay.
Let’s say that Fred is an athlete who needs to eat at least 2500 calories each
day, but that Fred can only spend $15 on food each day. To keep things simpler,
let’s just consider two types of food: healthy and unhealthy. Healthy food costs
$0.013 per calorie, and unhealthy food costs only $0.004 per calorie. If we let
x be the number of healthy calories Fred eats each day, and y the number of
unhealthy calories, then we have described a system of constraints. The caloric
intake constraint is x+y ≥ 2500, while the cost constraint is 0.013x+0.004y ≤ 15.
What does a viable solution mean in this case? A viable solution will satisfy
both inequalities. That means it will describe a possible way for Fred to eat that
day. We can check: can Fred afford to eat 1000 calories from healthy food and
1500 from unhealthy food? In other words, is (1000, 1500) a viable solution? The
answer is no, because even though the caloric intake is OK, the cost is too much.

(0.013)(1000) + (0.004)(1500) = 13 + 6 = 19 6≤ 15.

See Sample Problems for more examples.


4. Rearrange formulas to highlight a quantity of interest, using the same reasoning as in
solving equations. For example, rearrange Ohm’s law V = IR to highlight resistance
R.
What Every Math Teacher Should Know 53

– How do you rearrange a formula?


You can rearrange a formula by doing something algebraic to both sides of the
equation. You can add something to both sides, or subtract something from both
sides. You can multiply or divide both sides by something (as long as you do not
divide by zero). There are other things that you can do, but we will cover those
in the next section below.
– Isn’t this a lot like solving an equation?
Yes and no. Yes, it is a lot like solving an equation in that the algebraic steps are
the same. You are manipulating an equation using rules of algebra. But a formula
is usually a more general statement than an equation. As we will see, to solve an
equation means to find a specific value of a variable or all values of the variables
that satisfy the equation. When you solve for a variable in a formula, you are
usually just manipulating the other variable(s) in order to isolate the variable of
interest.
In their example, V = IR, you can highlight the resistance R by thinking of it as
a variable and thinking of the other letters as constants. Then IR is a lot like 3R
or 5R, and so we just need to divide by the coefficient. Since this is an equation,
we divide both sides by I to obtain
V
R= .
I
As long as I 6= 0, this equation is completely equivalent to the one we started
with, only now the variable representing resistance is by itself.
– Sample Problems
(a) The time it takes to drive 60 miles depends on how fast you are driving. Write
an equation describing this situation in terms of t, the time the journey takes,
and v, the speed at which you are driving. Sketch a graph of t as a function
of v.
(b) Create an equation for P and t, where P is the population of bacteria in
a Petri dish at time t. Assume the initial population is 1000 bacteria, and
that the population doubles every hour. Is this linear growth or exponential
growth? Explain.
(c) The amount of grass seed needed to cover a square field is proportional to
the area of the field. If G represents the amount of grass seed, and x is the
54 CCSS for High School Math: Algebra - by Christopher Goff, Ph.D.

length of the square field, write down an equation for this situation. (Use k
to represent the constant of proportionality.)
(d) The time, t, that it takes to grade papers depends on n, the number of papers
you have to grade. Assuming it takes 7 minutes to prepare the answer key
and then 3 minutes to grade each paper, write down an equation relating t
and n. Sketch a graph of t as a function of n.
(e) Let’s refine the previous problem about grading. Assume that your students
fall into two groups, the strong students and the struggling students. Suppose
you have b strong students and c struggling students. Now assume that it only
takes 2 minutes to grade a strong student paper, and 5 minutes to grade the
paper of a student who is struggling. Now write an equation relating t, b, and
c. (You still need 7 minutes to make an answer key.)
(f) Let’s return to Fred’s problem of taking in at least 2500 calories per day, but
only having $15 to spend on food. (See above. We assume that healthy food
costs $0.013 per calorie, while unhealthy food costs only $0.004 per calorie.)
Explain whether each of the following solutions (x, y) is viable, where x is the
number of healthy calories and y is the number of unhealthy calories.
i. (500, 1500)
ii. (500, 2000)
iii. (800, 1700)

(g) You may recall that the formula for a Celsius temperature C, is related to
its equivalent Fahrenheit temperature F via: C = 95 (F − 32). Manipulate
this equation so that the Fahrenheit temperature is isolated. What does the
formula tell us now?
(h) The area, A, of a circle depends on its radius, r: A = πr2 . Solve this equation
for r. What does the formula tell us now?

– Answers to Sample Problems


(a) The time it takes to drive 60 miles depends on how fast you are driving. Write
an equation describing this situation in terms of t, the time the journey takes,
and v, the speed at which you are driving. Sketch a graph of t as a function
of v.
There are a few ways to reach this answer. One is to recall that “rate times
What Every Math Teacher Should Know 55

60
time equals distance.” So vt = 60, or t =. Another way is to think about
v
a few points. How long would the journey take if you drove 60mph? One
hour. What if you drove 30mph? Two hours. What if you drove 120mph?
60
Then the journey would only take half an hour. So these also satisfy t = .
v

(b) Create an equation for P and t, where P is the population of bacteria in


a Petri dish at time t. Assume the initial population is 1000 bacteria, and
that the population doubles every hour. Is this linear growth or exponential
growth? Explain.
We’ll answer the second part first. This set-up is an example of exponen-
tial growth. Linear growth would increase by the same amount every hour,
whereas exponential growth increases by the same factor every hour. Let’s
compare these two models in a table, assuming each model starts with 1000
bacteria and has 2000 bacteria after one hour.

t (in hours) 0 1 2 3
Linear 1000 2000 3000 4000
Exponential 1000 2000 4000 8000

Now let’s find a formula for the exponential population, because that is the
one we want. First, let’s divide each entry by 1000 so we can see what is
going on. Then, our population is 1, 2, 4, and 8. Notice that these are powers
of 2: 20 , 21 , 22 , and 23 . In general, we get 2t . So the population satisfies
P = 1000 · 2t . (You could also think of it as a 100% growth rate each hour.
Then we would use r = 1 in the interest formula, P = 1000(1 + r)t , to get
P = 1000(1 + 1)t = 1000 · 2t .)
(c) The amount of grass seed needed to cover a square field is proportional to
the area of the field. If G represents the amount of grass seed, and x is the
length of the square field, write down an equation for this situation. (Use k
56 CCSS for High School Math: Algebra - by Christopher Goff, Ph.D.

to represent the constant of proportionality.) G = kx2 .


(d) The time, t, that it takes to grade papers depends on n, the number of papers
you have to grade. Assuming it takes 7 minutes to prepare the answer key
and then 3 minutes to grade each paper, write down an equation relating t
and n. Sketch a graph of t as a function of n.
t = 7 + 3n

(e) Let’s refine the previous problem about grading. Assume that your students
fall into two groups, the strong students and the struggling students. Suppose
you have b strong students and c struggling students. Now assume that it only
takes 2 minutes to grade a strong student paper, and 5 minutes to grade the
paper of a student who is struggling. Now write an equation relating t, b, and
c. (You still need 7 minutes to make an answer key.)
t = 7 + 2b + 5c
(f) Let’s return to Fred’s problem of taking in at least 2500 calories per day, but
only having $15 to spend on food. (See above. We assume that healthy food
costs $0.013 per calorie, while unhealthy food costs only $0.004 per calorie.)
Explain whether each of the following solutions (x, y) is viable, where x is the
number of healthy calories and y is the number of unhealthy calories.
i. (500, 1500) is NOT viable because x + y = 2000 6≥ 2500. Fred wouldn’t get
enough calories here.
ii. (500, 2000) is viable because x + y = 2500 and 0.013(500) + 0.004(2000) =
14.5 ≤ 15. Fred gets enough calories and stays under his budget.
iii. (800, 1700) is NOT viable because even though Fred would get enough
What Every Math Teacher Should Know 57

calories, he would go over his budget, since

0.013(800) + 0.004(1700) = 17.2 6≤ 15.

(g) You may recall that the formula for a Celsius temperature C, is related to
its equivalent Fahrenheit temperature F via: C = 95 (F − 32). Manipulate
this equation so that the Fahrenheit temperature is isolated. What does the
formula tell us now?
9
We would get F = 5
C + 32. Now the formula tells us which Fahrenheit
temperature corresponds to a given Celsius temperature. In other words, if
you know the temperature in degrees Celsius, then this formula is more useful
in finding the equivalent Fahrenheit temperature.
(h) The area, A, of a circle depends on its radius, r: A = πr2 . Solve this equation
for r. What does the
r formula tell us now?
A
We would get r = . This tells us the radius of a circle if we know its area.
π
(Since the radius of a circle cannot be negative, we do not use a ± sign.)
58 CCSS for High School Math: Algebra - by Christopher Goff, Ph.D.

REASONING WITH EQUATIONS


AND INEQUALITIES

• Understand solving equations as a process of reasoning and explain the


reasoning

1. Explain each step in solving a simple equation as following from the equality of numbers
asserted at the previous step, starting from the assumption that the original equation has
a solution. Construct a viable argument to justify a solution method.

– Why does algebra work?


Solving equations with variables is what most people think of when they think
of algebra. There are rules to how equations can be manipulated, and students
sometimes focus too much on the rules and lose sight of why the rules are true.
Most algebraic steps rely on the idea of equality, what it means for two things to
be exactly the same. If you think of numbers, a number can only equal itself - it
cannot equal a different number. So 4 = 4 is true, but 4 = 2 is false.
If two things are equal to begin with, and we transform both things in the exact
same way, then the two transformed things must still be equal. This is clear. If
you start with a true equality, like 4 = 4, and you add ten to both sides, then the
result (i.e. 14 = 14) must still be a true equality. That explains the vast majority
of algebraic manipulations of equations.
Inequalities are similar, but there are some transformations that will change the
direction of the inequality (namely, multiplying or dividing by a negative number),
and so one must be careful. If you start with a true inequality, like 4 > 2, and
you subtract 10 from both sides, you still get a true inequality: −6 > −8. But if
you now multiply both sides of this inequality by −1, then you must remember
to change the direction of the inequality. Namely, −6 > −8, but 6 < 8. (This
can be visualized on a number line. See Sample Problems, below.)
Understanding that we are simply maintaining relationships of equality or inequal-
ity throughout the algebraic steps will help students understand why algebra leads
toward a correct solution.
– What are some of the properties that equations and inequalities have?
What Every Math Teacher Should Know 59

Let’s start with equations. The main properties are: equality is reflexive (x = x),
symmetric (if x = y, then y = x), and transitive (if x = y and y = z, then x = z).
Also, there is an addition property of equality (if a = b then a + x = b + x,
or if a = b and x = y, then a + x = b + y) and a multiplication property of
equality (if a = b, then ax = bx, or if a = b and x = y, then ax = by). We don’t
list a separate subtraction property of equality or division property of equality
because subtraction is just addition of an additive inverse and division is just
multiplication by a multiplicative inverse. So both of those properties are already
covered by the ones we listed here.
Inequalities have similar properties. Both ≤ and ≥ are reflexive (x ≤ x and
x ≥ x are both true). None of the inequalities is symmetric. All of them are
transitive (e.g., if x < y and y < z, then x < z). There are various additive and
multiplicative properties of inequalities:
∗ If a < b and c < d, then a + c < b + d (and a + c < b + c).
∗ If a < b and c ≤ d, then a + c < b + d (and a + c < b + c).
∗ If a ≤ b and c ≤ d, then a + c ≤ b + d (and a + c ≤ b + c).
∗ If a < b and c > 0, then ac < bc.
∗ If a < b and c < 0, then ac > bc.
Combining these properties with other properties of numbers – additive and mul-
tiplicative identities and inverses; the associative and commutative properties of
addition and multiplication; and the distributive property of multiplication over
addition – should prove adequate to solve most any equation we see.
– Why can we assume that the original equation has a solution? Why does that
matter?
As we said before, if two things are equal to begin with, and we transform both
things in the exact same way, then the two transformed things must still be equal.
Notice the assumption that the two things were equal to begin with. If you start
with equal things, then any transformations must keep them equal.
A problem with this can arise because some transformations may take unequal
numbers and make them equal. For instance, it is clear that 4 6= −4. But if we
square both sides of this “equation,” then we get 16 = 16, a true statement. This
might lead students to think that they must have started with a true statement,
which is not logically correct. The multiplicative property of equality can be used
60 CCSS for High School Math: Algebra - by Christopher Goff, Ph.D.

to prove that if a = b, then a2 = b2 , but the converse is not true. Just because
a2 = b2 does not mean a = b.
So, we assume that the original equation has a solution. That is, we assume
there is some value of the variable making both sides equal. Then we take the
appropriate steps to finding that solution. Lastly, we have to check our answers
to make sure we didn’t introduce any incorrect solutions along the way.
– How do I know if a solution method is viable?
A solution method is viable as long as it doesn’t break any algebra rules, and finds
all valid solutions. Although, students tend to think of algebra as proscriptive -
that they have to follow the rules in a certain order or else they will get a wrong
answer. The good news is that as long as they are not breaking any algebra
rules, their equations should still be correct. The bad news is that often there is
a “best” path to a solution (or a few equivalently good routes) and so they might
be wandering for a long time unless they really learn how to manipulate equations
to their advantage.
2. Solve simple rational and radical equations in one variable, and give examples showing how
extraneous solutions may arise.
– How do you solve simple rational and radical equations in one variable?
The goal is to isolate the variable. Let’s look at two examples.
2
First, consider = 3. We can solve this equation by multiplying both sides by x
x
to obtain 2 = 3x. Now we divide both sides by 3 (or multiply both sides by 13 ) to
2 3
get x = 23 . We check this in the original equation: 2 = 2 · = 3, which is correct.
2
√ 3
Second, let’s try x − 5 = 6. Squaring both sides, we obtain x − 5 = 36 and so
√ √
x = 41. Checking this is the original equation, we get 41 − 5 = 36 = 6.
– What is an extraneous solution?
An extraneous solution is a solution to a transformed equation that is not a
solution to the original equation. We did not obtain any extraneous solutions
in the first two examples because we checked our answers and they satisfied the
original equations.
– Why do extraneous solutions arise?
Extraneous solutions can arise whenever your algebraic step is not reversible. We
talked earlier about how 4 = −4 is a FALSE equation, but the transformed equa-
tion from squaring both sides (16 = 16) is true. That’s essentially how extraneous
What Every Math Teacher Should Know 61

solutions can arise. Squaring both sides of an equation is not a reversible step.
Neither is multiplying both sides by zero. One normally wouldn’t multiply both
sides of an equation by zero on purpose, but when variables are involved, stu-
dents sometimes forget to check if they have inadvertently introduced extraneous
solutions.
– What are some examples of extraneous solutions?
Let’s do some examples in which extraneous solutions arise. First, consider the

equation 4 − 3x2 = x. Squaring both sides, we get 4−3x2 = x2 , which simplifies
to 4 = 4x2 or x2 = 1. We’ll talk about quadratic equations more below, but for
now we will point out that there are two solutions: x = 1 or x = −1 (often written
x = ±1).
If we check these values in the original equation, it’s clear that x = 1 is a valid
solution, but x = −1 is extraneous:

p √ √
4 − 3(−1)2 = 4 − 3 = 1 = 1 6= −1.

Squaring both sides of the original equation introduced an extraneous solution.


2x 4
As a second example, consider = 3+ . To clear the denominators, we
x−2 x−2
will multiply both sides by x − 2. So we get

2x = 3(x − 2) + 4
2x = 3x − 6 + 4
2x = 3x − 2
2 = x.

So x = 2. But if we try to check this value in the original equation, we get a


denominator of zero. So x = 2 is an extraneous solution and there is no solution
to the original equation. Here, we introduced the extraneous solution when we
multiplied both sides by x − 2. This step is only reversible if x − 2 6= 0, i.e., if
x 6= 2. If x = 2, it can still be done, but it destroys your chances of finding any
other solutions. It’s like trying to solve x + 1 = 2 by multiplying both sides by
zero. You just get 0 = 0, which is true, but you have lost any other information
from the original equation.
62 CCSS for High School Math: Algebra - by Christopher Goff, Ph.D.

• Solve equations and inequalities in one variable


3. Solve linear equations and inequalities in one variable, including equations with coefficients
represented by letters.
– What does it mean to solve an equation? . . . an inequality?
To solve an equation (or an inequality) means to find all the values of the variable
that make the equation (or inequality) into a true statement. This is accomplished
by following the rules of algebra and using the various properties of numbers and
of equality (or inequality) as needed.
– Can you explain why the solution to 3x − 5 = 4 is x = 3 by showing each step
and listing all the properties you used?
Being an algebraist, I like to show all the properties that are being used in the
solution. Admittedly, it might seem like overkill, and you might not require your
students to show this level of detail, but I find it fascinating that so much structure
underlies a simple act like solving a linear equation.

3x − 5 = 4 Given
(3x − 5) + 5 = 4 + 5 Additive Property of Equality
(3x − 5) + 5 = 9 Arithmetic
(3x + (−5)) + 5 = 9 Definition of Subtraction
3x + ((−5) + 5) = 9 Associative Property of Addition
3x + 0 = 9 Additive Inverse
3x = 9 Additive Identity
1 1
(3x) = (9) Multiplicative Property of Equality
3 3
1
(3x) = 3 Arithmetic
 3 
1
·3 x=3 Associative Property of Multiplication
3
1x = 3 Multiplicative Inverse
x=3 Multiplicative Identity

I should explain that the reasons listed above describe the transformation that
occurred between the previous equation and the current one. So, when I list
Additive Inverse above, it does NOT mean that 3x+0 = 9 because of the additive
What Every Math Teacher Should Know 63

inverse property. Rather, it means that the previous equation 3x + ((−5) + 5) = 9


transforms into the equation 3x + 0 = 9 because of the additive inverse property.
So it really just explains why −5 + 5 = 0.
– Can you explain why the solution to −3x − 5 < 4 is x > −3 by showing each step
and listing all the properties you used?

−3x − 5 < 4 Given


(−3x − 5) + 5 < 4 + 5 Additive Property of Inequality
(−3x − 5) + 5 < 9 Arithmetic
(−3x + (−5)) + 5 < 9 Definition of Subtraction
−3x + ((−5) + 5) < 9 Associative Property of Addition
−3x + 0 < 9 Additive Inverse
−3x < 9 Additive Identity
1 1
− (−3x) > − (9) Multiplicative Property of Inequality
3 3
1
− (−3x) > −3 Arithmetic
 3 
1
− · −3 x > −3 Associative Property of Multiplication
3
1x > −3 Multiplicative Inverse
x > −3 Multiplicative Identity

– What if there are coefficients that are letters?


The process is the same. You just have to remember which letters are coefficients
and which ones are the variables. So for instance, in solving Ax + B = C, we
can subtract B from both sides, giving Ax = C − B. Now we divide by A to get
C −B
x= . We should make sure that A 6= 0 (or else we couldn’t divide by it).
A
But if A = 0, then the original equation is just B = C, and there are no variables
at all! So it’s probably safe to assume that A 6= 0 here.
As another example, we could solve the equation V = IR for R. So we would
V
get R = . Notice that this is exactly what we had to do earlier when we were
I
manipulating a formula to highlight one of the variables in the formula. The
techniques are the same, but the interpretation may be different. The power of
algebra may begin with its techniques, but it is greatly amplified by the different
64 CCSS for High School Math: Algebra - by Christopher Goff, Ph.D.

levels at which those techniques can be interpreted.

4. Solve quadratic equations in one variable.

(a) Use the method of completing the square to transform any quadratic equation in x
into an equation of the form (x − p)2 = q that has the same solutions. Derive the
quadratic formula from this form.

– How does completing the square enable you to put the equation in the form
(x − p)2 = q?
As we said before on page 15, “One way to interpret completing the square
is that we are making a substitution that removes the linear term.” So if we
have an equation, we can put it into a form like ay 2 = p, or after dividing by
a, y 2 = ap . This kind of an equation will be much easier to solve.
As an example, let’s look at 2x2 + 5x + 5 = 10. If we complete the square the
way we did earlier, we would get

 
2 5 25 5 25
2 x + x+ + − = 10
2 16 2 16
 2 !
5 15
2 x+ + = 10
4 16
 2
5 15
2 x+ + = 10
4 8
 2
5 15 65
2 x+ = 10 − =
4 8 8
 2
5 65
x+ = ,
4 16

which is in the desired form. We will solve equations of this form below.
Now that we are dealing with equations instead of just expressions, we can
complete the square another (equivalent) way. We can just add the same
thing to both sides of the equation. Let’s start again with 2x2 + 5x + 5 = 10,
25
or 2x2 + 5x = 5. Notice that we begin by adding 8
to both sides, but it just
looks different on the left hand side due to the parentheses and the factor of
What Every Math Teacher Should Know 65

2 in front.
 
2 5 25 25
2 x + x+ = 5+
2 16 8
 2
5 65
2 x+ =
4 8
 2
5 65
x+ = ,
4 16

which agrees with what we found before. Some people prefer this approach
to completing the square over the first one. As long as you don’t break any
algebra rules, either solution method will work.
– How do you derive the Quadratic Formula?
We will outline the steps below. Note where we complete the square, because
after that step, the equation is in (x − p)2 = q form.

ax2 + bx + c = 0 Given (a 6= 0)
b c
x2 + x = − Divide by a 6= 0 and rearrange terms
a a
2 b b2 c b2
x + x+ 2 =− + 2 Complete the square
a 4a a 4a
2
b2 − 4ac

b
x+ = Factor, obtain common denominator
2a 4a2
  √
b b2 − 4ac
x+ =± Take square root of each side, with ±
2a 2a

b b2 − 4ac
x=− ± Rearrange terms
2a 2a

−b ± b2 − 4ac
and thus x = .
2a
(b) Solve quadratic equations by inspection (e.g., for x2 = 49), taking square roots,
completing the square, the quadratic formula and factoring, as appropriate to the
initial form of the equation. Recognize when the quadratic formula gives complex
solutions and write them as a ± bi for real numbers a and b.
– How can you solve a quadratic equation by inspection or by taking square
roots?
The method of inspection is easiest to use when the equation is of the form

x2 = A. If A > 0, then there are two real solutions: x = ± A. If A = 0, then
there is only one solution: x = 0, and if A < 0, then there are two imaginary
66 CCSS for High School Math: Algebra - by Christopher Goff, Ph.D.

√ √
solutions: x = ± i −A, where i = −1. (Note that in this last case, −A is
a positive number.)
The method of taking square roots could be used when there is only a slight
variation to the above form, like (x − p)2 = q. Then taking square roots of
√ √
both sides (and remembering the ±), we get x − p = ± q, and so x = p ± q.
Again, there are two real solutions if q > 0, one solution if q = 0, and two
complex solutions if q < 0.

One example: solve (x + 5)2 + 1 = 3. Then (x + 5)2 = 2, and so x + 5 = ± 2

and therefore x = −5 ± 2.
– How do you solve a quadratic equation by completing the square?
Recall that completing the square puts a quadratic equation into the form
(x−p)2 = q. From here, we can simply take square roots to solve the equation.
One example: x2 + 4x + 9 = 36. So,

x2 + 4x = 27
x2 + 4x + 4 = 27 + 4 = 31
(x + 2)2 = 31,

which means that x = −2 ± 31.
– How do you solve a quadratic equation by factoring?
Sometimes a quadratic equation can be factored. This means that it can be
put into the form a(x − r)(x − s) = 0. From here, we can use the converse of
the multiplication property of zero: if a product of numbers is equal to zero,
then at least one of the factors has to equal zero. So that would mean that
x − r = 0 (i.e., x = r) or x − s = 0 (i.e., x = s). This can be an extremely
useful way to solve quadratic equations.
For example, consider x2 + 3x + 9 = 6 − x. First we gather all the terms on
one side, setting the quadratic equal to zero. So we obtain: x2 + 4x + 3 = 0,
which factors as (x + 1)(x + 3) = 0. The solution to the original problem is
thus x = −1 or x = −3.
– How can you tell which method to use?
This is the tricky part. If there is no linear term, then I would solve the
equation by inspection or by doing some rearranging of terms and then taking
square roots. If there is a linear term, then I first try gathering terms and
What Every Math Teacher Should Know 67

factoring. If the quadratic doesn’t factor, then I will complete the square or
use the quadratic formula.
– When does a quadratic equation have complex roots? How can you tell? How
can you find them?
A quadratic equation always has complex roots. (Remember, real numbers are
also complex.) But I think that the standard is referring to complex solutions
that are not real. If the equation is of the form ax2 + bx + c = 0, then
the discriminant b2 − 4ac (the expression under the radical in the quadratic
formula) determines what kind of roots you have. If b2 − 4ac < 0, then there
are two (non-real) complex roots, and they can be found using the quadratic
formula. If b2 − 4ac = 0, then there is one real root. If b2 − 4ac > 0, then
there are two real roots. And if b2 − 4ac happens to be a perfect square, then
the original equation could have been solved by factoring.
Consider x2 + x + 1 = 0. The discriminant is (1)2 − 4(1)(1) = −3 < 0. So
there are two complex roots. They are:
√ √
−1 ± −3 −1 3
= ± i .
2 2 2
Some books teach how to factor a sum of two squares using complex numbers.
Since x2 − y 2 = (x − y)(x + y), and since one can write x2 + y 2 as x2 − i2 y 2 ,
then the sum of squares factors as (x − iy)(x + iy). This can allow you to
find some complex roots without using the quadratic formula. For instance,
if x2 + 9 = 0, then (x − 3i)(x + 3i) = 0, and so x = ±3i are the two complex
(in this case, imaginary) roots.

– Sample Problems
(a) Solve the following equations. Identify any extraneous solutions, if they arise.
i. 2x − 5 = 16.
ii. 2x − 5 > 16.
iii. bx − c = 16.
x
iv. = 2.
x−2
x
v. = M.
x−1
x
vi. ≤ 2. (Careful!)
x−2
68 CCSS for High School Math: Algebra - by Christopher Goff, Ph.D.

x 3
vii. =3− .
x+3√ x+3

viii. x + x + 3 = 3.

ix. x2 + 2x + 1 = x + 1 (Careful!)
(b) Show on a number line that if a > b > 0, then −a < −b.
(c) Let a and b be integers with b 6= 0. Consider the following statement: If
a
b
< 1, then a < b.
i. List some values for a and b that make the statement true.
ii. List some values for a and b that make the statement false.
iii. What is a condition on a and/or b that will make the statement necessarily
true?
(d) Solve the following quadratic equations.
i. 2x2 = 20.
ii. 3(x + 4)2 = 12.
iii. x2 − 6 = 5x.
iv. x2 + 6 = 5x.
v. x2 + 6x = 5.
vi. 3x2 + 6x = 5.
(e) Use the quadratic formula to solve the following. Leave constants in your
answer, if necessary. If the answers are complex, write them in a + bi form.
i. x2 + bx + 1 = 0.
ii. x2 − x + 1 = 0.
1
iii. − At2 + V t + S = 0.
2
(f) Let f (x) = x2 − bx + (b − 1). Find f (1). Explain how the Factor Theorem
allows you to factor f (x). Then, factor f (x).
– Answers to Sample Problems
(a) Solve the following equations. Identify any extraneous solutions, if they arise.
21
i. 2x − 5 = 16. x = = 10.5
2
21
ii. 2x − 5 > 16. x > = 10.5
2
16 + c
iii. bx − c = 16. x = , assuming b 6= 0.
b
What Every Math Teacher Should Know 69

x
iv. = 2. x = 4
x−2
x M
v. = M. x = , assuming M 6= 1.
x−1 M −1
Here’s one solution:
x
= M
x−1
x = M (x − 1) = M x − M
x − M x = −M
x(1 − M ) = −M
−M M
x = = .
1−M M −1
x
vi. ≤ 2. (Careful!) x ≥ 4 or x < 2.
x−2
To see this, let’s rewrite the left hand side by finding quotient and remainder
polynomials first. It will simplify things a little bit by getting the variable
x in only one location.
x x−2+2 2
2≥ = =1+ ,
x−2 x−2 x−2
2
and so 1 ≥ x−2
. Now we need to multiply by x − 2 but whether or not we
change the direction of the inequality depends on the value of x. So let’s
look at two cases: x − 2 < 0 (i.e., x < 2) and x − 2 > 0 (i.e., x > 2). (Notice
that if x = 2 we have an undefined fraction.)
Case 1: x − 2 < 0. Multiplying by x − 2, we get x − 2 ≤ 2. Since x − 2
is negative in this case, it is certainly less than 2. Every value of x in this
case solves the inequality.
Case 2: x − 2 > 0. Multiplying by x − 2, we get x − 2 ≥ 2, or x ≥ 4. So
not every value of x in this case will work. We also have to restrict x to be
at least 4.
Putting the two cases together, we get that x < 2 or x ≥ 4.
x 3
vii. = 3− . The “solution” x = −3 is extraneous; there are no
x+3 x+3
solutions.
√ √
viii. x + x + 3 = 3. x = 1

ix. x2 + 2x + 1 = x + 1 (Careful!) This looks like an identity once you square
both sides. It is tempting to state that the equation is true for all values

of x. But it’s not. Try x = −2 for instance. 4 − 4 + 1 = 1 6= −2 + 1.
70 CCSS for High School Math: Algebra - by Christopher Goff, Ph.D.

The equation is only true if the right-hand side is not negative. That is,
x + 1 ≥ 0, or x ≥ −1. To reiterate: this equation is true for all values of x
that are greater than −1.
(b) Show on a number line that if a > b > 0, then −a < −b.
We are told that a > b > 0. On a number line, this looks like:
0 b a

So, if we put in −a and −b as well, we get:


−a −b 0 b a

Clearly, −a is to the left of −b, and thus −a < −b.


(c) Let a and b be integers with b 6= 0. Consider the following statement: If
a
b
< 1, then a < b.
i. List some values for a and b that make the statement true. Answers may
vary, although b must be greater than zero.
ii. List some values for a and b that make the statement false. Answers may
vary, although b must be less than zero.
iii. What is a condition on a and/or b that will make the statement necessarily
true? If b is positive, then one can use the Multiplicative Property of
Inequality to deduce that the statement must be true. Conversely, if b < 0,
then the statement is false.
(d) Solve the following quadratic equations.

i. 2x2 = 20. x = ± 10 (I used inspection.)
ii. 3(x + 4)2 = 12. x = −4 ± 2, i.e. x = −2 or x = −6. (I used inspection.)
iii. x2 − 6 = 5x. x = 6 or x = −1. (I factored.)
iv. x2 + 6 = 5x. x = 2 or x = 3. (I factored.)

v. x2 + 6x = 5. x = −3 ± 14. (I completed the square.)
q
vi. 3x2 + 6x = 5. x = −1 ± 83 . (I completed the square.)
(e) Use the quadratic formula to solve the following. Leave constants in your
answer, if necessary. If the answers are complex, write them in a + bi form.

−b ± b2 − 4
i. x2 + bx + 1 = 0. x =
√ 2 √
2 1 ± −3 1 3
ii. x − x + 1 = 0. x = = ±i
2 2 2
What Every Math Teacher Should Know 71


1 2 V ± V 2 + 2AS
iii. − At + V t + S = 0. x =
2 A
2
(f) Let f (x) = x − bx + (b − 1). Find f (1). Explain how the Factor Theorem
allows you to factor f (x). Then, factor f (x).
f (1) = 1 − b + (b − 1) = 0. The Factor Theorem implies that (x − 1) is thus
a factor of f (x). So

x2 − bx + (b − 1) = (x − 1)(x − (b − 1)).

• Solve systems of equations

5. Prove that, given a system of two equations in two variables, replacing one equation by
the sum of that equation and a multiple of the other produces a system with the same
solutions.

– How do we know that if we change the equations that make up a system, we are
not also changing the solutions to that system?
When working with a system of equations, it is important that you not change the
equations in such a way that you wind up changing the solutions of the original
system. This standard describes one way to ensure that the original solutions are
not altered.
Let’s use some symbols to see the operation that the standard is describing.
Suppose your system consists of two equations: A = B and C = D. The standard
claims that the solution to this system is the same as the solution to the system
of two equations: A + nC = B + nD and C = D, where n is any number. Why is
this true? The answer hinges on the fact that the Addition Property of Equality
is reversible. That is, one can also “add” −n times the second equation to the
new first equation, which gives a system: A + nC − nC = B + nD − nD (i.e.,
A = B) and C = D, which is the original system. So we can return to where we
started.
Compare that with multiplication. If we were to multiply the first equation by n,
giving us a system: nA = nB and C = D, then we might not be able to reverse
the process. This would happen if n = 0. We would have a true equation, but we
would have lost part of the information that the original system contained.
– What are some other “valid” moves that also do not change the solution set to a
system of equations?
72 CCSS for High School Math: Algebra - by Christopher Goff, Ph.D.

In light of the previous paragraph, one can multiply an equation in a system by


a nonzero constant without changing the solutions to the system. One can also
switch the order of the equations without changing their solution set.
One common technique to use is substitution, which is a good technique for
reducing the number of distinct equations in the system. The only caveat is
to make sure that one is not over-substituting, by plugging information from
Equation 1 into Equation 2, say, and then plugging information from the new
Equation 2 back into Equation 1. Often this leads to circular reasoning and little
progress towards finding a solution. We will explore some examples in the next
section.
6. Solve systems of linear equations exactly and approximately (e.g., with graphs), focusing
on pairs of linear equations in two variables.
– How do you find exact solutions?
The main methods have been mentioned above: replacing one equation by its
sum with a multiple of another equation, replacing one equation by a nonzero
multiple of itself, switching the order of equations, and substitution. If a system
can be solved exactly, it will usually be by a combination of these approaches.
– How do you find approximate solutions?
Approximate solutions can be found by graphing, or by using technology, such
as a graphing calculator or other mathematical software. Sketching the graphs of
the equations that make up the system allows one to see if the individual graphs
intersect. The intersection points are the solutions to the system. For more on
how to use graphing to solve equations, see page 88, and the Sample Problems
below. To use a graphing calculator or other software, consult the manual or
online help features.
– What are some examples?
We’ll look at two ways to solve the system the equations: 2x + 3y = 5 and
x − y = 5.
Method 1 (manipulating equations): By looking at the left-hand sides, we
see a 3y term in the first equation and a −y term in the second equation. So let’s
add 3 times the second equation to the first equation. This gives

2x + 3y + 3(x − y) = 5 + 3(5) and x − y = 5 or:


5x = 20 and x − y = 5.
What Every Math Teacher Should Know 73

From here, we can divide the first equation by 5 (or multiply it by 15 ). So the
equations now read x = 4 and x − y = 5. We can now substitute to find that
y = −1. The solution to the system is the single point (4, −1).
Method 2 (substitution): First, we will rewrite the second equation to solve for
x, obtaining x = y + 5. We now substitute this expression into the first equation
for x to obtain:

2(y + 5) + 3y = 5 ⇐⇒ 2y + 10 + 3y = 5 ⇐⇒ 5y = −5.
1
From here, we can multiply the equation by 5
(or divide by 5) to obtain y = −1.
Then, since x = y + 5, x = 4. Therefore, (x, y) = (4, −1) is the solution.
Next, let’s solve the following system of equations: 4x − 3y = 15 and 6x + y = 6.
As before, let’s add 3 times the second equation to the first. This makes the first
equation into 4x − 3y + 3(6x + y) = 15 + 3(6), or after simplification, 22x = 33,
3
which means x = 2
= 1.5. From here, we can use either equation to determine
the value of y. Using the second equation, we get 6(1.5) + y = 6, or 9 + y = 6,
from which we get y = −3. So x = 1.5 and y = −3.
Next, let’s solve the following system of equations: x − y = 12 and −3x + 3y = 3.
Dividing the second equation by −3 (or multiplying by − 13 ) gives x − y = −1.
So our system is now x − y = 12 and x − y = −1. Hence there are no solutions
to this system of equations. Such a system is called inconsistent. If we had
tried to add 3 times the first equation to the second, we would have obtained
−3x + 3y + 3(x − y) = 3 + 3(12), or 0 = 39, which is clearly not a true statement.
That is a sure sign of an inconsistent system.
And as a final example, let’s solve the system x − 5y = 2 and −2x + 10y = −4.
Here, let’s add 2 times the first equation to the second, to obtain:

−2x + 10y + 2(x − 5y) = −4 + 2(2),

or 0 = 0. What does this mean?


It means that the second equation is a multiple of the first. Thus, we really
only need one of the equations. This system has an infinite number of solutions,
namely any pair (x, y) that satisfies x − 5y = 2 will necessarily satisfy the system.
Here, the original equations are said to be dependent on each other.
7. Solve a simple system consisting of a linear equation and a quadratic equation in two
variables algebraically and graphically. For example, find the points of intersection between
the line y = −3x and the circle x2 + y 2 = 3.
74 CCSS for High School Math: Algebra - by Christopher Goff, Ph.D.

– How do you find exact solutions?


The main methods from before are still viable: replacing one equation by its sum
with a multiple of another equation, replacing one equation by a nonzero multiple
of itself, switching the order of equations, and substitution. When nonlinear
equations are involved in the system, substitution may play a more active role
than the other methods.

– How do you solve their example exactly?


The standard mentions the system: y = −3x and x2 +y 2 = 3. To solve this exactly,
we will begin by using substitution since the first equation is already solved for y.
we get x2 +(−3x)
Substituting −3x for y in the second equation, r √
2
= 3, or 10x2 = 3,
3 3 30
which means x2 = . By inspection, x = ± =± . Remembering that
10 √ 10 10
3 30
y = −3x, we get y = ∓ . So there are two solutions:
10

√ √ ! √ √ !
30 3 30 30 3 30
,− and − , .
10 10 10 10

– How do you find approximate solutions?


Also as before, approximate solutions can be found by graphing, or by using
technology, such as a graphing calculator or other mathematical software. For
more on how to use graphing to solve equations, see page 88. To use a graphing
calculator or other software, consult the manual or online help features.

– How do you find approximate solutions to their example?


If we were to find approximate solutions, though, then we could do that by graph-
ing the line y = −3x and the circle x2 + y 2 = 3. Since we know that the line
passes through the origin with negative slope, and the circle is centered at the
origin, then we will have two intersection points, one in Quadrant II and one in
Quadrant IV. Moreover, the intersection points will be endpoints of a diameter
of the circle. To find approximate values, we will plot the curves on the same set
of axes and look for intersection points.
What Every Math Teacher Should Know 75

r points are about (−0.55, 1.65) and (0.55, −1.65).


It looks like the intersection
3
That’s very close, because ≈ 0.54772, and the y-value is −3 times that, or
10
about −1.64317.

– What are some other examples?


Let’s find the intersection between x = 4 + y and y = x2 − 4x. The problem seems
well suited to substitution. We can either substitute 4 + y for x in the quadratic
equation, or we can substitute x2 −4x for y in the linear equation. I will choose the
latter, just because there is less algebra involved. So the first equation becomes
x = 4+(x2 −4x), which can be rearranged to 0 = x2 −5x+4 = (x−4)(x−1). There
are two solutions: x = 1 and x = 4. If x = 1, then y = (1)2 − 4(1) = 1 − 4 = −3.
If x = 4, then y = (4)2 − 4(4) = 0. So the two solutions to the system are (1, −3)
and (4, 0). Notice that we kept the corresponding x and y values together in
each solution. The point (4, −3) is not a solution to either of the two original
equations.
To solve this system graphically, see the graph below.
76 CCSS for High School Math: Algebra - by Christopher Goff, Ph.D.

The solutions are clearly (1, −3) and (4, 0).


– Sample Problems
(a) Solve the following systems exactly.
i. x + y = 4 and x − 2y = 3.
ii. 4x + 5y = 30 and 2x − 3y = 20.
iii. y = x + 4 and 2x − 2y = 8.
iv. y = x + 4 and 2x − 2y = −8.
(b) Graph the following systems, which we examined earlier. Connect the solution
sets to what is happening graphically.
i. 2x + 3y = 5 and x − y = 5
ii. 4x − 3y = 15 and 6x + y = 6
iii. x − y = 12 and −3x + 3y = 3
iv. x − 5y = 2 and −2x + 10y = −4
(c) Solve the following systems exactly (using algebra) and approximately (using
a graph).
i. y = 6 − x and y = x2
ii. x2 + y 2 = 9 and 2y = x

– Answers to Sample Problems


(a) Solve the following systems exactly.
What Every Math Teacher Should Know 77

 
11 1
i. x + y = 4 and x − 2y = 3. (x, y) = ,
3 3
 
95 10
ii. 4x + 5y = 30 and 2x − 3y = 20. (x, y) = ,−
11 11
iii. y = x + 4 and 2x − 2y = 8. No solution. The system is inconsistent.
iv. y = x + 4 and 2x − 2y = −8. The two equations are multiples of each
other. The solution set is any point (x, y) satisfying y = x + 4.

(b) Graph the following systems, which we examined earlier. Connect the solution
sets to what is happening graphically.
i. 2x + 3y = 5 and x − y = 5

The two lines intersect in the single point (4, −1), which is the solution.
ii. 4x − 3y = 15 and 6x + y = 6

The two lines intersect in the single point (1.5, −3), which is the solution.
iii. x − y = 12 and −3x + 3y = 3
78 CCSS for High School Math: Algebra - by Christopher Goff, Ph.D.

The two lines are parallel and therefore have no intersection point, which
corresponds to the inconsistency of the system.
iv. x − 5y = 2 and −2x + 10y = −4

The two lines are identical, and so any point on the line(s) is a solution to
the system.
(c) Solve the following systems exactly (using algebra) and approximately (using
a graph).
i. y = 6 − x and y = x2 (−3, 9) and (2, 4)

√ √ ! √ √ !
6 5 3 5 6 5 3 5
ii. x2 + y 2 = 9 and 2y = x , and − ,−
5 5 5 5
What Every Math Teacher Should Know 79

From the graph, we can tell that the solutions are approximately (2.7, 1.35)
and (−2.7, −1.35).

8. (+) Represent a system of linear equations as a single matrix equation in a vector variable.
9. (+) Find the inverse of a matrix if it exists and use it to solve systems of linear equations
(using technology for matrices of dimension 3 × 3 or greater).

We will treat these two standards together, since we will first remind you of what
a matrix is and how to multiply them and find determinants. Then we will be
able to write a system of linear equations as a single matrix equation with a vector
variable. We will also show how matrices can be used to solve systems of linear
equations, and not only using inverse matrices, but also using row operations.
– What is a matrix?
A matrix is a rectangular array of numbers. Matrices can be very useful in solving
systems of linear equations, among other applications.
– How do you multiply matrices?
You can multiply two matrices if the number of columns of the first matrix equals
the number of rows of the second matrix. As an example,
" #" # " #
a b 1 2 3 a + 4b 2a + 5b 3a + 6b
= .
c d 4 5 6 c + 4d 2c + 5d 3c + 6d

In general, if A is an m by n matrix and B is an n by p matrix, then AB is an m


by p matrix and the entry of AB in row i and column j is given by:
n
X
(AB)ij = Aik Bkj .
k=1
80 CCSS for High School Math: Algebra - by Christopher Goff, Ph.D.

– How does matrix multiplication help you write a system of linear equations as a
single matrix equation with a vector variable?
Given the way matrix multiplication works, it is well suited to writing systems of
linear equations in a compact form. It’s perhaps best to look at some examples.
Consider the system x + 3y = 7 and 4x − 4y = 2. Compare this with the matrix
equation " #" # " #
1 3 x 7
= .
4 −4 y 2
Notice that if we perform matrix multiplication on the left-hand side, we get
" # " #
x + 3y 7
= ,
4x − 4y 2
which is completely equivalent to the starting system. So if we can solve the
matrix equation, we can solve the original system also. Notice too that both
variables x and y lie in a single 2 by 1 matrix. This can be thought of as a
two-dimensional vector variable. Vectors play a large role in physics and higher
mathematics, and so it’s good to get used to this way of “seeing” a system of
equations.
One more example: let’s convert the system x − 2y = 10 and −4x + 5y = −15
into a single matrix equation.
" #" # " #
1 −2 x 10
= .
−4 5 y −15
– What is the determinant of a matrix? How does the determinant of a matrix
relate to its invertibility?
The determinant of a square matrix is a specific number that encodes some of the
properties of that matrix. For instance, if det M = 0, then the matrix M is not
invertible. If det M 6= 0, then there is a matrix N satisfying M N = N M = I,
where I means the identity matrix (1s on the diagonal, 0s elsewhere). In this
case, N is also called M −1 , the inverse matrix of M .
– How do you calculate the determinant? How do you calculate the inverse of a
matrix?
For 2 by 2 matrices, the determinant is given by:
" #
a b a b
det = = ad − bc.

c d c d
What Every Math Teacher Should Know 81

Also, for 2 by 2 matrices, there is a relatively simple formula for finding the inverse
matrix:
" #−1 " # " #
d −b
d −b

a b 1 ad−bc ad−bc
= = .
c d ad − bc −c a −c a
ad−bc ad−bc

Notice that the matrix in invertible if (and only if) its determinant, ad − bc is not
equal to zero. That’s why it’s okay for us to divide by ad − bc throughout.
For 3 by 3 matrices, the determinant is given by:

a b c


d e f = aei + bf g + cdh − bdi − af h − ceg.


g h i
One way to remember this formula involves recopying the first two columns and
then looking along the diagonals of the resulting array.
a b c a b
d e f d e
g h i g h
Multiplying along diagonals down and to the right, we get the terms aei, bf g,
and cdh. These are the first three (positive) terms in the determinant formula.
Multiplying down and to the left, we obtain the terms bdi, af h, and ceg, which
are the next three (negative) terms in the determinant formula.
There is also a recursive way to find the determinant, called expansion by minors.
This means that the determinant of a 3 by 3 matrix (for instance) can be written
in terms of determinants of various 2 by 2 submatrices of the original matrix.
The tricky part is that there is a factor of (−1)r+c , where r is the row number
and c the column number, counted from the upper left. We’ll expand along the
top row, although any row or column would work. Pick the first element, a, and
then form a submatrix by deleting the row and column containing a. Continue
throughout the row. See the example, below.

a b c

e f d f
4 d e

d e f = (−1) a 2 3
+ (−1) b + (−1) c





h i g i g h
g h i
= a(ei − f h) − b(di − f g) + c(dh − eg)
= aei + bf g + cdh − af h − bdi − ceg.
82 CCSS for High School Math: Algebra - by Christopher Goff, Ph.D.

Expansion by minors applies to larger matrices, whereas the trick of repeating


the first two columns works only for 3 by 3 matrices.
As for finding the inverse of a 3 by 3 matrix or 4 by 4 or higher, the Common
Core Standards recommend using technology. While the topic is usually covered
in a college Linear Algebra course, technology is indeed helpful, as it cuts down on
the number of arithmetic computations involved. Many graphing calculators will
invert small matrices, but for more advanced matrix inverses, use mathematical
software.

– How can you use a matrix to determine the solvability of a system of linear
equations?
If you have a system of n linear equations in n variables, you can write it as
A~x = ~b, where A is the square (n by n) coefficient matrix, ~x is the vector of
variables, and ~b is a column vector of right-hand sides to the equations. See
the examples above. Method 1 below uses row operations, which are really just
manipulations of entire equations. For instance, you can multiply an equation by
a nonzero constant. So, one valid row operation is to multiply the entire row by
a nonzero constant.
If A is invertible (that is, if det A 6= 0), then there is exactly one solution to the
system, namely ~x = A−1~b. Again, see Example, below, Method 2.
If A is not invertible, then the situation is a little trickier. There might be no
solutions (in which case the system of equations is inconsistent), or there might
be an infinite number of solutions. We have seen examples of these above, on
page 73.

– How can you use a matrix to solve a system of linear equations?

Example: Solve the equations 2x + 3y = 5 and x − y = 5 simultaneously.

Method 1 (row operations):


We start by making an augmented matrix, which includes the coefficients of the
variables, but also includes the right-hand sides.
What Every Math Teacher Should Know 83

" # " #
2 3 5 1 −1 5
∼ (switch rows)
1 −1 5 2 3 5
" #
1 −1 5
∼ (add −2(row 1) to row 2)
0 5 −5
" #
1 −1 5
∼ (divide row 2 by 5)
0 1 −1

So the equations now read x − y = 5 and y = −1. We can substitute to find that
x = 4.

Method 2 (inverse matrices): First, rewrite the system in matrix form:


" #" # " #
2 3 x 5
= .
1 −1 y 5
Using the formula for the inverse of a 2 by 2 matrix gives:
" #−1 " # " #
−1 −3

2 3 1 1/5 3/5
= = .
1 −1 (2)(−1) − (3)(1) −1 2 1/5 −2/5

So, we can multiply the original matrix equation on the left to obtain:
" #" #" # " #" #
1/5 3/5 2 3 x 1/5 3/5 5
= ,
1/5 −2/5 1 −1 y 1/5 −2/5 5

which simplifies to
" #" # " # " #
1 0 x x 4
= = .
0 1 y y −1

Therefore, (x, y) = (4, −1).


– Sample Problems
(a) Check the formula for the 2 by 2 inverse matrix by calculating AA−1 and
A−1 A.
(b) Give an example of 2 by 2 matrices A and B satisfying AB 6= BA.
(c) Solve the following system of equations using the inverse matrix: 4x−3y = 15
and 6x + y = 6.
84 CCSS for High School Math: Algebra - by Christopher Goff, Ph.D.

(d) Solve the following system. You may use the technology of your choice.

5x + 3y − 10z = 56
7x − y − 4z = 43
−12x − 15y + 20z = −140
" #
11 6
(e) Find the determinant of .
2 −5
 
4 3 7
 
(f) Find the determinant of 
 5 −5 4 .

0 −9 −8
" # " #
3 5 9
(g) Find B so that AB = C, where A = and C = .
−3 4 9
(h) Using A and C above, find AC, if possible. Then find CA, if possible.

– Answers to Sample Problems


(a) Check the formula for the 2 by 2 inverse matrix by calculating AA−1 and
A−1 A. We assume ad − bc 6= 0 so that the inverse of A is defined.
" # " #! " # " #
a b 1 d −b 1 ad − bc −ab + ba 1 0
= = ,
c d ad − bc −c a ad − bc cd − dc −cb + da 0 1
" #! " # " # " #
1 d −b a b 1 ad − bc db − bd 1 0
= = .
ad − bc −c a c d ad − bc −ca + ac −bc + ad 0 1
(b) Give an example of 2 by 2 matrices A and B satisfying AB 6= BA. There are
many answers.
" #" # " # " #" # " #
0 1 0 0 1 0 0 0 0 1 0 0
= , but = .
0 0 1 0 0 0 1 0 0 0 0 1

(c) Solve the following system of equations using the inverse matrix: 4x−3y = 15
and 6x + y = 6.
Using matrices, we get
" #" # " #
4 −3 x 15
= .
6 1 y 6
What Every Math Teacher Should Know 85

" #
1 1 3
The inverse of the coefficient matrix is . So, multiplying both
22 −6 4
sides (on the left) by this inverse matrix gives:
" #" #" # " #" #
1 1 3 4 −3 x 1 1 3 15
=
22 −6 4 6 1 y 22 −6 4 6
" # " # " #
x 1 33 1.5
= = .
y 22 −66 −3

So x = 1.5 and y = −3.


(d) Solve the following system. You may use the technology of your choice.

5x + 3y − 10z = 56
7x − y − 4z = 43
−12x − 15y + 20z = −140

We can set up a matrix equation and then use row operations or the inverse
matrix to solve the system. The corresponding matrix equation is:
    
5 3 −10 x 56
    
 7 −1 −4  y  =  43 .
    
−12 −15 20 z −140

Using technology (TI-83 Plus), I found the determinant of the coefficient


matrix A to be 494, and so
 
−80 90 −22
1 
A−1 =

 −92 −20 −50 
494  
−117 39 −26

So, the answer is:


       
x 56 2470 5
−1 = 1 
       
 y =A  43 988  =  2 .
    494    
z −140 −1235 −2.5

5
Therefore, x = 5, y = 2, and z = − .
2
86 CCSS for High School Math: Algebra - by Christopher Goff, Ph.D.

" #
11 6
(e) Find the determinant of . −67
2 −5
 
4 3 7
 
(f) Find the determinant of 
 5 −5 4 . 109

0 −9 −8
" # " #
3 5 9
(g) Find B so that AB = C, where A = and C = . In order for
−3 4 9
" #
x
AB to be a 2 by 1 matrix, we need B to be a 2 by 1 matrix. Let B =
y
−1 −1
and solve. Or, find A . Then B = A C. In any case,
" #
− 13
B= .
2

(h) Using
" #A and C above, find AC, if possible. Then find CA, if possible. AC =
72
. The product CA is not defined because C has only one column, but
9
A has two rows.

• Represent and solve equations and inequalities graphically


10. Understand that the graph of an equation in two variables is the set of all its solutions
plotted in the coordinate plane, often forming a curve (which could be a line).
– What does a graph of an equation in two variables really represent?
Graphing is a powerful tool. It provides a way for us to see equations and to
study them by looking at the shapes of their solutions. The power comes from
the fact that each point on a graph of an equation of two variables corresponds
to a solution of that equation. And conversely, each solution to the equation
corresponds to a point on the graph.
For example, let’s consider the graph of y = x. This equation has an infinite
number of solutions, but every solution is a pair of x and y values that are equal,
such as (0, 0), (1, 1), (−6, −6), (− 35 , − 53 ), etc. Plotting all of these points on
the same set of coordinate axes starts to reveal a line. If you think about it,
every point on that line, even ones with irrational coordinates, will still satisfy
the equation y = x. That straight line represents the equation y = x in a visual
way.
What Every Math Teacher Should Know 87

What about y = x2 ? This is also an equation in two variables, and it also has
an infinite number of solutions, but for every solution, the y value is equal to the

square of the x-value. So, points like (0, 0), (1, 1), (−1, 1), (−2, 4), and ( 2, 2)
are on this curve. If you plot these points, you find that the graph is curved into
a shape called a parabola, with a vertex at the origin. And every point on that
parabola is a solution to the equation y = x2 for some value of x.

One more example: x2 + y 2 = 25. Here, we also have an infinite number of


√ √
solutions, like (5, 0), (0, −5), (−3, 4), (3, 4), ( 11, 14), etc. Plotting these points
(or recognizing the distance formula) shows that this equation describes a circle of
radius 5 centered at the origin. Every solution to the equation lies on this circle,
and every point on the circle is a solution to the equation.
88 CCSS for High School Math: Algebra - by Christopher Goff, Ph.D.

Using imprecise language, we sometimes say that this equation “is” a circle. It
would be better to say that the graph of the equation is a circle, or that the
equation can be represented graphically by a circle.
– How can a curve be a line?
Here, the term “curve” is used in a general way, meaning pretty much any path
(curvy or straight) that you can draw with your pencil. So, a “curve” can be
truly curved, or it could be straight, or it could even have parts that are straight
and other parts that are curved.
11. Explain why the x-coordinates of the points where the graphs of the equations y = f (x)
and y = g(x) intersect are the solutions of the equation f (x) = g(x); find the solutions
approximately, e.g., using technology to graph the functions, make tables of values, or find
successive approximations. Include cases where f (x) and/or g(x) are linear, polynomial,
rational, absolute value, exponential, and logarithmic functions.**
– Why are the x-coordinates of the points where the graphs of the equations y =
f (x) and y = g(x) intersect the solutions of the equation f (x) = g(x)?
This has to do with the fundamental meaning of what the points on a graph
represent. We know that the points on the graph of y = f (x) are all the solutions
to this equation. That is, a point on this graph has the form (a, f (a)) for some
value of a. Similarly, each point on the graph of y = g(x) has the form (b, g(b))
for some value of b.
What if there is a point that lies on both graphs? Let’s say it has an x-coordinate
of c. Because it lies in the first graph, it has the form (c, f (c)), and because it
lies on the second graph, it has the form (c, g(c)). But it’s the same point, so
(c, f (c)) = (c, g(c)), which means that for this value c, f (c) = g(c). Therefore, c
What Every Math Teacher Should Know 89

is a solution to the equation f (x) = g(x).


This argument is reversible. Suppose that there is a solution d to the equation
f (x) = g(x). Then f (d) = g(d). That means that the point (d, f (d)) = (d, g(d))
lies on both graphs, and hence must be a point of intersection.
As an example, let’s consider some simple functions: f (x) = x and g(x) = x2 .
Then this standard is saying that the solutions to x = x2 are exactly the x-
coordinates of the intersection points of the graphs of y = x and y = x2 . Let’s
check this.
First, let’s find the solutions to x = x2 . This quadratic can be rearranged and
factored, giving: 0 = x2 − x = x(x − 1), which has two solutions: x = 0 and
x = 1.
Second, let’s look at the two graphs: y = x and y = x2 . We just graphed these
in the previous section. The graph of y = x is a line passing through the origin,
while y = x2 is a parabola with a vertex at the origin. Graphing them on the
same axes, we get

From this, it is clear that there are only two points of intersection, corresponding
to x = 0 and x = 1. So yes, the solutions to x = x2 are exactly the same as the
x-coordinates of the intersection points of y = x and y = x2 .
This is not surprising. After all, we just proved it before doing this example. But it
does open up a new way to solve equations, one that relies on visual evidence more
than algebraic manipulations. For many students (especially for visual learners),
this can be a powerful way to access their understanding of algebra.

– How do we approximate these solutions?


Graphing calculators, mathematical software, and even some websites will find
approximate solutions to intersections of graphs. Consult colleagues or the manual
90 CCSS for High School Math: Algebra - by Christopher Goff, Ph.D.

to figure out how to do it on your device or using your program.


As an example of how technology can be helpful, find the intersection point be-
tween y = x5 and y = 12 x + 3. We will graph them below.

The graph makes it clear that there is only one intersection point, and it looks
like this point is located at approximately (1.3, 3.7). Using software to find the
solution to x5 = 12 x + 3, I get an approximate solution to be 1.295.
– How do you find approximate solutions using a table of values? Why use a table?
Tables are nice because if students are good with numbers, then they can some-
times see a concept with a table much better than they can see it on a graph or
with an equation. I’ll describe one way below, though there are certainly many
others.
Let’s continue with the equation x5 = 12 x+3, but let’s rewrite it as x5 − 12 x−3 = 0.
So a solution will be a zero of the polynomial x5 − 21 x − 3. Let’s look at values of
this polynomial in a table.

x 1.1 1.2 1.3 1.4


1
x5 − 2
x − 3 −1.9395 −1.1117 0.0629 1.6782

Since polynomials are continuous, the change of sign means that there must be
a zero between 1.2 and 1.3. It looks close to 1.3. Let’s calculate values that are
“zoomed in,” and let’s carry some more decimal places in our answer.

x 1.28 1.29 1.30


x5 − 12 x − 3 −0.20403 −0.07269 0.06293
What Every Math Teacher Should Know 91

So now we know the zero is between 1.29 and 1.30. Let’s zoom in again.

x 1.294 1.295 1.296


5 1
x − 2
x − 3 −0.01897 −0.00543 0.00816

Now we know the zero is between 1.295 and 1.296. We could continue zooming
in until we get as many decimal places as we wish.
– Sample Problems
For each equation, approximate the solutions by graphing, and then verify your
guesses using tables of values or software. You may use the technology of your
choice.
(a) 5x + 14 = −2x − 5.
x2
(b) 2 = 2x + 1.
x +1
(c) log x = −x + 4.
(d) x4 = 12 (x + 5). (Find all solutions.)
(e) |3 − x2 | = 5x . (Find all solutions.)
– Answers to Sample Problems
For each equation, approximate the solutions by graphing, and then verify your
guesses using tables of values or software. You may use the technology of your
choice.
(a) 5x + 14 = −2x − 5. x ≈ −2.7143

x −2.713 −2.714 −2.715


7x + 19 0.009 0.002 −0.005
x2
(b) = 2x + 1. x ≈ −0.42385
x2 + 1
92 CCSS for High School Math: Algebra - by Christopher Goff, Ph.D.

x −0.4238 −0.4239 −0.4240


2
x
− 2x − 1 −0.000140 0.000121 0.000381
x2 +1

(c) log x = −x + 4. x ≈ 3.4608

x 3.459 3.460 3.461


log x + x − 4 −0.002049 −0.000924 0.000202

(d) x4 = 12 (x + 5). (Find all solutions.) x ≈ −1.1759, 1.3340


What Every Math Teacher Should Know 93

x −1.175 −1.176 −1.177


4 1
x − 2
(x + 5) −0.006375 0.000623 0.007636

x 1.333 1.334 1.335


x4 − 12 (x + 5) −0.009165 −0.000180 0.008826

(e) |3 − x2 | = 5x . (Find all solutions.) x ≈ −1.7493, −1.7136, 0.6025


In the graph below, the roughly “W”-shaped graph is the graph of y = |3−x2 |.

x −1.748 −1.749 −1.750


|3 − x2 | − 5x −0.004503 −0.000909 0.002686

x −1.712 −1.713 −1.714


|3 − x2 | − 5x 0.005470 0.002147 −0.001178

x 0.6023 0.6024 0.6025


2 x
|3 − x | − 5 0.000966 0.000421 −0.000123
94 CCSS for High School Math: Algebra - by Christopher Goff, Ph.D.

12. Graph the solutions to a linear inequality in two variables as a half-plane (excluding the
boundary in the case of a strict inequality), and graph the solution set to a system of linear
inequalities in two variables as the intersection of the corresponding half-planes.
– Why is the graph of the solution to a linear inequality a half plane?
If you can solve the linear inequality for y, then it is clear that you are looking
for values of y either above (y > f (x)) or below (y < f (x)) the line. If y doesn’t
appear in the equation, then the line must be vertical, and the inequality tells
you if you are looking for points to the right (x > a) or to the left (x < a) of this
line.
– Why is the boundary excluded if the inequality is strict?
A strict inequality is one in which equality is not allowed (like < or >). Since
the boundary line is all the points where equality holds, the points lying on the
boundary line would not satisfy a strict inequality. In the following examples, the
boundary lines have been labeled, and dotted boundary lines are not part of the
solution set.
Examples: y ≤ 5, x > −2, x + 2y ≥ 3
x = -2

y=5

y = (-x+3)/2

– How do you graph the solution to a system of linear inequalities?


A good way to see how to graph a system of linear inequalities is via a common
application: linear programming. Linear programming can be used to solve op-
timization problems in many different fields. Usually, you are asked to maximize
some quantity with respect to various linear inequality constraints.
Simple(x) Example: Say you have 20 days to knit hats and scarves for a
friend’s store. It takes you 1.5 days to knit a hat and only 1 day to knit a scarf.
You plan to charge $20 per hat and $15 per scarf, but your friend says that she
wants no more than 16 items from you. How many hats and how many scarves
should you knit in order to maximize your revenue?
What Every Math Teacher Should Know 95

Answer: Let x be the number of hats knitted and y the number of scarves
knitted. So x ≥ 0 and y ≥ 0. Also, x + y ≤ 16 because your friend only wants 16
items at most. The number of days it takes to knit hats is 1.5x, while the number
of days it takes to knit scarves is y. So 1.5x + y ≤ 20 since there are only 20 days
to knit. If we graph all of these inequalities, we obtain a region of all the possible
numbers of scarves and hats you could knit. The revenue function is 20x + 15y,
which we would like to maximize on the given region. According to the simplex
method, since the revenue condition is linear, we need only check the corners of
our region, which occur at any intersection point of two linear conditions.

y = 20 - 1.5x

(0,16)

(8,8)

y = 16 - x

(0,0) (13.3,0)

Checking, we get:
∗ no scarves and no hats yields 0 dollars of revenue
∗ 16 scarves and no hats yields (16)(15) = 240 dollars of revenue
∗ 8 scarves and 8 hats yields 8(15) + 8(20) = 280 dollars of revenue
∗ 13 hats and no scarves yields 13(20) = 260 dollars of revenue
So, to maximize revenue, you should knit eight scarves and eight hats.
– Sample Problems
(a) Sketch the solution to y ≤ 2x − 5.
(b) Sketch the solution to 2x + 3y > 6.
(c) Sketch all the points (a, b) with a ≤ 2b.
(d) Express the points inside Quadrant II as the solution to a system of strict
linear inequalities.
(e) Suppose that a company makes two kinds of puzzles: easy and hard. The
company has 10 weeks to make puzzles before putting the products on the
96 CCSS for High School Math: Algebra - by Christopher Goff, Ph.D.

market. They can make 60 easy puzzles per week or 40 hard puzzles per week.
They make $12 profit on each easy puzzle and $15 profit on each hard puzzle.
Assuming that they can only put 500 puzzles on the market, how many of
each should they make?
– Answers to Sample Problems
(a) Sketch the solution to y ≤ 2x − 5.
y = 2x - 5

(b) Sketch the solution to 2x + 3y > 6.

y = 2 - 2x/3

(c) Sketch all the points (a, b) with a ≤ 2b.


b

a = 2b

(d) Express the points inside Quadrant II as the solution to a system of strict
linear inequalities. The system is: x < 0 and y > 0.
(e) Suppose that a company makes two kinds of puzzles: easy and hard. The
company has 10 weeks to make puzzles before putting the products on the
market. They can make 60 easy puzzles per week or 40 hard puzzles per week.
They make $12 profit on each easy puzzle and $15 profit on each hard puzzle.
Assuming that they can only put 500 puzzles on the market, how many of
What Every Math Teacher Should Know 97

each should they make? 300 easy, 200 hard.


The inequalities come from different constraints. Let x be the number of easy
puzzles made and y the number of hard puzzles made. We know x ≥ 0 and
y ≥ 0. Also, x+y ≤ 500 because they can only put 500 puzzles on the market.
1
They only have 10 weeks to make puzzles. Since it takes 60
of a week to make
1 x y
an easy puzzle and 40
of a week to make a hard puzzle, 60 + 40 ≤ 10, or, if
we multiply both sides of the inequality by 120, 2x + 3y ≤ 1200. These are
graphed below.
y

500 x + y = 500

400

(300, 200)

2x + 3y = 1200

500 x

The corners of the region of interest are (0, 0), (0, 400), (500, 0), and (300, 200),
where x is the number of easy puzzles made and y is the number of hard
puzzles made. Checking each one, we obtain the maximum profit ($6600) at
(300, 200).

You might also like